Search This Blog

Thursday, January 14, 2010

MCCQE 1

Community and Public Health


1) Which one of the following is not a typical feature of asbestosis?

a) increased risk of cancer

b) pleural thickening and calcification

c) interstitial fibrosis

d) obstructive pattern on pulmonary function tests

e) none of the above

2) The following statements regarding contact dermatitis are true

EXCEPT:

a) phototoxic dermatitis following topical application of

creosote requires UV light

b) photoallergic contact dermatitis requires UV light to be

manifested

c) contact eczema involves a type IV delayed

hypersensitivity reaction

d) contact urticaria or hives is a common form of dermatitis

e) chemical burns by HCl and KOH may result in an irritant

contact dermatitis

3) The following statements regarding noise are true EXCEPT:

a) temporary threshold shift recovers following cessation of

noise exposure

b) permanent threshold shift is characterized by a

progressive pattern of hearing loss

c) most cases of permanent threshold shift are surgically treatable

d) higher frequency noise is more damaging than low frequency

noise

e) none of the above

4) The frequencies most necessary for the understanding of

speech extend from about:

a) 20-20 000 Hz

b) 400-4 000 Hz

c) 250-8 000 Hz

d) 100-5 000 Hz

e) none of the above

5) Lead exposure typically results in:

a) chronic dermatitis

b) resting and intention tremor

c) extensor muscle weakness

d) arrhythmias

e) cerebellar ataxia

6) Which of the following statements concerning the Worker’s

Compensation Act is true?

a) the worker reserves the right to sue the employer for negligence

b) funding is provided by the provincial government

c) the worker is guaranteed payment from the first day of

injury/illness if it is deemed to be work-related

d) the Worker’s Compensation Board is an independent, private

agency

e) none of the above

7) Which of following statements regarding radiation is false?

a) natural background radiation accounts for about half of a

typical person’s exposure

b) ionizing radiation causes intestinal villi to become denuded

c) exposure to non-ionizing radiation may result in cataracts

d) ionizing radiation results in an increased incidence of

neoplasia such as lung and thyroid

e) none of the above

8) Which statement concerning vibration induced white

finger disease is false?

a) early symptoms include tingling and numbness of the

fingers when at rest

b) swelling of the fingers over knuckles may be an early

feature

c) cold, damp conditions may precipitate symptoms

d) the affected area eventually spreads to involve all fingers

e) none of the above

9) In a cohort study of disease “X” in people with risk factor

“Y” versus those who are without risk factor “Y”, the

following results were obtained:

X no X total

Y 80 20 100

no Y 50 50 100

The relative risk of developing “X” in “Y” versus no “Y” is:

a) 80 x 50 = 4 c) 50 x 80 = 4

50 x 20 20 x 50

b) 50 _ 20 = 0.3 d) 80/100 = 1.6

100 100 50/100

e) it is not possible to calculate risk in a cohort study

10) The attributable risk of factor “Y” to disease “X” would be:

a) 80/100 = 1.6 c) 80 x 50 = 4

50/100 50 x 20

b) 80 _ 50 = 0.3 d) 50 x 20 = 0.25

100 100 80 x 50

e) it is not possible to calculate atrributable risk

in a cohort study

11) Agroup of 50 people are exposed to virus “A”. Of those 50

people, 9 develop a mild infection, 10 become seriously ill, and 3

die. The attack rate of virus “A” in the population would be:

a) 22/50

b) 9/50

c) 10/50

d) 19/50

e) 13/50

12) The following indicate the results of screening test “Q” in

screening for disease “Z”:

Disease Z

+ –

+ 40 10 50

Screen Q – 30 120 150

70 130 200

The specificity of test “Q” would be:

a) 40/70

b) 120/130

c) 40/50

d) 120/150

e) 40/130

13) The positive predictive value would be:

a) 40/70

b) 120/130

c) 40/50

d) 120/150

e) 70/200

14) To determine an odds ratio one would have to perform

which of the following studies?

a) a cross sectional/prevalence study

b) a randomized controlled trial

c) a cohort study

d) a case study

e) a case control study

Toronto Notes 2005 Community and Public Health Sample Questions 3

15) Examples of secondary prevention would include all of the

following EXCEPT:

a) Pap smear for cervical cancer

b) chemoprophylaxis in a recent TB converter

c) proctoscopy for rectal cancer

d) immunization for Haemophilus influenzae B

e) mammography for breast cancer

16) Alpha error is:

a) the probability of declaring a difference to be absent

when it in fact is present

b) the probability of declaring a difference to be present

when it is not

c) the probability of declaring a difference to be absent

when it is indeed absent

d) the probability of declaring a difference to be present

when it does exist

17) Which one of the following descriptors of a diagnostic test is

influenced by the prevalence of the disease being tested for:

a) specificity

b) sensitivity

c) accuracy

d) positive predictive value

e) reliability

18) Which of the following statements regarding the measurement

of health and disease in a population is true?

a) a rate is the number of times an event has occured

during a certain time interval divided by the number of

persons at risk during the same interval

b) when calculating a ratio, the numerator is a portion of

the denominator

c) a ratio is the number of times an event has occurred

during a certain time interval multiplied by the number

of times an event has occurred during the same interval

d) when calculating a rate, the numerator is not a portion of

the denominator

e) none of the above

19) In 1990, which country spent the least on health care as a

percentage of GDP (gross domestic product):

a) Canada

b) France

c) Sweden

d) UK

e) USA

20) The component of Canada's health care system that

receives the highest percentage of the health care budget is:

a) hospitals

b) physician fees

c) drug benefit plans

d) laboratory services

e) administration

21) The British North America Act (BNA):

a) was mainly concerned with health care legislation

b) granted exclusive powers over health care to the

provinces, including powers over marine hospitals

c) replaced the Canada Health Act

d) applied to British Columbia only

e) none of the above

22) Regarding health expenditure and health outcomes:

a) the U.S. has the greatest health expenditure and the

lowest infant mortality rates

b) there is a positive association between national

expenditure on health and GDP

c) increased national health expenditure always increases

health status of a country

d) all of the above

e) none of the above

23) Which of the following is the most important justification for

population screening programs for a specific disease?

a) early detection of the disease of interest is achieved

b) the specificity of the screening test is high

c) the natural history of the disease is favourably altered by

early detection

d) effective treatment is available

e) the screening technology is available

24) Regarding the regulation of health professionals, provincial

colleges of physicians and surgeons:

a) have the advancement of the public interest as their

primary goal

b) protect the public from incompetent or unfit MDs

c) act as licensing bodies for MDs

d) do not advance the professional and political interests of MDs

e) all of the above

25) Active immunization was important in control of each of the

following childhood communicable diseases EXCEPT:

a) diphtheria

b) polio

c) measles

d) scarlet fever

e) pertussis

26) All of the following statements are true EXCEPT:

a) one indirect measure of a population’s health status is the

percentage of low birth weight neonates

b) accidents are the largest cause of potential years of life lost

in Canada

c) the Canadian population is steadily undergoing

rectangularization of mortality

d) morbidity is defined as all health outcomes excluding death

e) the neonatal mortality rate is the number of infant deaths

divided by the number of live births multiplied by 1000

27) All of the following statements are true EXCEPT:

a) the data collected on a death certificate is uniform and in

conformity with WHO guidelines

b) Section 7 of the Coroner’s Act states that the coroner’s office

must be notified if a patient dies after some mishap such

as leaving an instrument in the body at surgery

c) diseases which must be reported to the local medical

officer of health include AIDS, food poisoning, influenza

and gonorrhea

d) a tuberculin reaction greater than 5 mm is considered

positive in all individuals

e) all of the above

28) In describing the leading causes of death in Canada, two very

different lists emerge, depending on whether proportional

mortality rates or person-years of life lost (PYLL) are used. This

is because:

a) one measure uses a calendar year and the other a fiscal

year to calculate annual experience

b) one measure includes morbidity as well as mortality

experience

c) both rates exclude deaths occurring over the age of 70

d) different definitions of “cause of death” are used

e) one measure gives greater weight to deaths occurring in

younger age groups

29) Differentiation between a point-source epidemic and a

progressive (propagated) epidemic is made by:

a) considering the characteristics of the infectious agent

b) determining the level of immunity in the community

c) determining the number of persons infected and

calculating the attack rate

d) plotting the distribution of cases by time onset

e) none of the above

38) In 1981, the crude birth rate in Ontario was approximately

14 per 1000 and the crude death rate was 7 per 1000. The

estimated rate of net migration was –1 per 1000. The growth

rate of the province, per 1000 population was:

a) 6

b) 7

c) 8

d) 20

e) 22

39) All of the following statements about environmental

health are true EXCEPT:

a) levels of toxic agents measured in the environment may not

reflect internal organ levels

b) the federal government monitors the quality and types of

industrial emissions and toxic waste disposal

c) sick building syndrome is associated with Pontiac fever and

Legionnaire’s disease

d) all humans have detectable levels of PCBs

e) none of the above is true

40) The effectiveness of a preventative measure is assessed in terms of:

a) the effect in people to whom the measure is offered

b) the effect in people who comply with the measure

c) availability with the optimal use of resources

d) the cost in dollars versus the benefits in improved health status

e) all of the above

41) All of the following statements about the Canada Health

Act (1984) are true EXCEPT:

a) it did not define all medically necessary hospital and

physician services

b) the CHA replaced the Hospital Insurance and Diagnostic

Services Act of 1957

c) the CHA banned all forms of extra billing

d) according to the CHA, provinces must meet all the terms

and conditions of Medicare to qualify for federal transfer

payments

e) none of the above

42) Each of the following is an example of primary prevention EXCEPT:

a) genetic counselling of parents with one retarded child

b) nutritional supplements in pregnancy

c) immunization against tetanus

d) chemoprophylaxis in a recent tuberculin converter

e) speed limits on highways

43) The classical “epidemiological triad” of disease causation consists of

factors which fall into which of the following categories:

a) host, reservoir, environment

b) host, vector, environment

c) host, agent, environment

d) reservoir, agent, vector

e) host, age, environment

44) Of the five items listed below, the one which provides the strongest

evidence for causality in an observed association between exposure

and disease is:

a) a large attributable risk

b) a large relative risk

c) a small p-value

d) a positive result from a cohort study

e) a case report

45) The difference between a common point source outbreak and

a propagated outbreak of illness is that:

a) all cases in a common point source outbreak occur within

one incubation period of the exposure

b) the attack rates in propagated outbreaks are higher

c) person-to-person transmission is a feature of common

source outbreaks

d) case fatality rates in common source outbreaks are generally

higher

e) the source of infection in propagated outbreaks is more

easily contained than in common source outbreaks

30) The occurrence of an illness at a rate of above that expected is

called:

a) hyperendemic

b) epidemic

c) endemic

d) enzootic

e) pandemic

31) Each of the following statements applies to case control

studies EXCEPT:

a) starts with disease

b) suitable for rare diseases

c) relatively inexpensive

d) prolonged follow-up required

e) there may be a problem in selecting and matching controls

32) Aclinician who has been examining the patterns of mortality in

your community says that the rates for heart disease and lung

cancer are higher in this community than in an adjacent

community. Which of the following questions should you ask first?

a) how did the clinician choose the comparison community?

b) have the rates been standardized for age?

c) are tobacco sales significantly different in the two communities?

d) are the facilities to treat these diseases comparable in the

two areas?

e) are the numbers of deaths comparable in each area?

33) The purpose of randomization is to:

a) make sure that there are equal numbers of men and women

in test and control groups

b) increase the chances of getting a statistically significant difference

c) ensure that the numbers of cases and controls are equal

d) limit bias

e) all of the above

34) Which of the following types of studies usually provides only a

measure of prevalence?

a) descriptive

b) cross-sectional

c) randomized controlled trial

d) cohort

e) none of the above

35) The major advantage of cohort studies over case-control

studies is that:

a) they take less time and are less costly

b) they can utilize a more representative population

c) it is easier to obtain controls who are not exposed to the factor

d) they permit estimation of risk of disease in those exposed

to the factor

e) they can be done on a “double-blind” basis

36) All of the following statements concerning occupational

health are true EXCEPT:

a) disorders of reproduction are among the top 10

work-related diseases and injuries

b) most workers are covered by both federal and provincial

legislation with respect to workplace health and safety

c) skin problems and hearing problems together are

responsible for half of WCB claims

d) a complete occupational medical history includes

investigation of the temporal relationship between

symptoms and exposure

37) Which of the following statements concerning exposure to solvents

in the workplace is true?

a) each solvent compound has a specific antidote that can be

used to treat exposure

b) a prominent symptom of solvent exposure is memory loss

c) some solvents can cause skin dryness and loss of

subcutaneous adipose tissue

d) solvents do not affect the bone marrow

e) all of the above

4 Sample Questions Community and Public Health Toronto Notes 2005

Toronto Notes 2005 Community and Public Health Sample Questions 5

46) Which of the following are strategies for control of disease:

a) population immunization

b) contact tracing to offer treatment to all who could be infected

c) monitoring increases in the population of certain disease vectors

d) having physicians report curable, potentially serious diseases

e) all of the above are true

47) During a clinical trial, the difference in the success rates of two

drugs was not statistically significant. This means that:

a) there is no difference in drug effectiveness

b) there is a sizable probability that the demonstrated difference in

the drugs’ effectiveness could occur due to chance alone

c) the demonstrated difference in the drugs’ effectiveness is

too small to be clinically meaningful

d) the two samples of patients on which the drugs were tested

came from the same population

e) none of the above are true

48) The incidence of a particular disease is greater in men than in

women, but the prevalence shows no sex difference. The most

probable explanation is that:

a) the mortality rate is greater in women

b) the case fatality rate is higher in women

c) the duration of the disease is longer in women

d) women receive less adequate medical care for the disease

e) this diagnosis is more often missed in women

49) All of the following statements about statistical tests are true

EXCEPT:

a) linear regression is used to describe the relationship

between two continuous variables

b) a confidence interval is a range of values giving information

about the precision of a measurement

c) ANOVAtables are used to make comparisons among the means

of 3 or more groups simultaneously

d) in a normal distribution, the mean, median and mode are equal

e) the chi-square test evaluates the statistical significance of 2

or more percentages of categorical outcomes

50) All of the following are responsibilities of local public health

units in Canada EXCEPT:

a) communicable disease control

b) health education

c) investigation of sudden death

d) immunization

e) health promotion

51) Who is ultimately responsible for the quality of care in a hospital?

a) chief executive officer

b) board of trustees

c) medical director

d) chief of staff

e) attending physicians

52) Which of the following is not one of the 5 Terms and

Conditions of Medicare?

a) portability

b) flexibility

c) universality

d) comprehensive coverage

e) accessibility

53) You are given this data pertaining to an outbreak of diarrhea

in a daycare:

Age Number of Children Number of Diarrhea

1 20 17

2 19 15

3 39 13

4 39 4

5 38 5

6+ 18 1

What is the attack rate of this illness?

a) 25% d) 40%

b) 30% e) 50%

c) 32%

6 Sample Questions Medicine Toronto Notes 2005

9) Hemolytic anemia is characterized by all of the following EXCEPT:

a) increased LDH

b) increased reticulocytosis

c) increased unconjugated bilirubin

d) increased haptoglobin

e) lead poisoning

10) Apatient complains of a non-tender mass over the thyroid region

on the left side of her neck. Concerned about a thyroid disorder,

you order the appropriate investigations. The results are as

follows:

TSH: 6.0

Free T4: 20.2

Thyroid antibodies: none

RAIU: No “hot” spots seen

The next investigation(s) you choose to do are:

a) watch and wait for 3-6 months

b) FNA

c) surgical biopsy

d) trial of L-thyroxine therapy for 6 months

e) none of the above

11) An 8 year-old boy is brought to the office because his mother is

concerned he is entering puberty already. You examine him and

note the beginnings of facial hair, axillary hair and Tanner stage 2

external genitalia. Choose the set of investigations you initially

want to do:

a) CBC, lytes, testosterone, bone age, CT head

b) FSH, LH, testosterone, lytes, bone age, DHEA-S

c) FSH, LH, testosterone, cortisol, DHEA-S, 11-OH progesterone,

bone age

d) lytes, testosterone, DHEA-S, 17-OH progesterone, cortisol, bone

age

12) The subendothelium is the most vulnerable segment of the heart

from an ischemic standpoint. The major reason for this is:

a) the highest oxygen utilization is in the subendocardium

b) coronary flow to the subendocardium occurs almost completely

during diastole whereas other regions receive some flow

during systole as well

c) the subendocardium has a diminished aerobic capacity

d) there is less potential for collateralization to the

subendocardium

e) the ratio of capillary to myocyte is less in the subendocardium

than in other regions

13) 18 year-old female with initial onset of pruritic rash characterized

by excoriations, scaling and crusting and distributed on the

extremities, neck and eyelids. Past medical history is significant for

asthma and hayfever. The most likely diagnosis is:

a) scabies

b) atopic dermatitis

c) contact dermatitis

d) shingles

e) dyshydrotic eczema

14) 8 year-old black male comes in with an asymptomatic

erythematous eruption characterized by oval patches with collarette

scaling. It is distributed as a "Christmas tree" pattern on the back.

Father states that there was originally one lesion on the abdomen a

few weeks prior. What is the most likely diagnosis:

a) pityriasis rosea

b) tinea corporis

c) lichen planus

d) psoriasis

e) none of the above

Medicine

1) Which of the following is a feature of secretory diarrhea?

a) small stool volume (< 1L/day)

b) increased stool osmotic gap

c) persistent diarrhea despite fasting

d) blood and/or pus in stools

e) malodorous, often floating stools

2) Each of the following are risk factors for colon cancer EXCEPT:

a) low fiber diet

b) severe diverticular disease

c) familial adenomatous polyposis

d) ulcerative colitis

e) high fat diet

3) Intestinal complications more common to Crohn’s disease than

ulcerative colitis include each of the following EXCEPT:

a) fistula formation

b) perianal disease

c) intestinal obstruction

d) toxic megacolon

e) post-surgical recurrence

4) A21 year-old bisexual man has a 4 week history of intermittent

diarrhea, urethral discharge, and pain in the right knee and left

second toe. He has several oral ulcers, a clear urethral discharge, a

scaly papular rash on palms and soles, onycholysis, sausage-like

swelling of the left second toe, and heat and swelling of the right

knee. The results of Gram stains and cultures of urethral discharge

are negative. Rheumatoid factor is not present. The most likely

diagnosis is:

a) Reiter’s syndrome

b) gonococcal arthritis

c) Behcet disease

d) acquired immune deficiency syndrome

e) psoriatic arthritis

5) A53 year-old presents to your office with pain and stiffness in

both hands and knees of 6 months duration. Which of the

following findings on your physical examination may help with a

diagnosis:

a) joint tenderness/effusions

b) maculopapular rash

c) iridocyclitis

d) hepatosplenomegaly

e) all of the above

6) Radiographic features of osteoarthritis of the knee include

which of the following:

a) marginal erosions

b) juxta-articular osteopenia (demineralization)

c) loss of articular cartilage with narrowing of the radiologic

joint space

d) osteonecrosis (avascular necrosis) of the medial femoral

condyle

e) high riding patella (patella alta)

7) An elevated level of hemoglobin A2 in a patient with mild

microcytic anemia suggests the diagnosis of:

a) alpha-thalassemia

b) sickle trait

c) beta-thalassemia

d) hereditary spherocytosis

e) hereditary persistence of fetal hemoglobin

8) It is unlikely to see macrocytosis in a patient with anemia in which

of the following?

a) reticulocytosis

b) vitamin B12 deficiency

c) folate deficiency

d) myelodysplastic syndrome

e) sideroblastic anemia

Toronto Notes 2005 Medicine Sample Questions 7

15) 30 year-old female comes in with a soft smooth erythematous

nodule on her lower lip. She states that a few weeks prior she had

some chapped lips with occasional bleeding. Now, the lips have

healed but this lesion arose suddenly in its place. It is occasionally

tender on pressure. The most likely diagnosis is:

a) HSV1

b) cherry hemangioma

c) pyogenic granuloma

d) dermal nevus

e) none of the above

16) A65 year-old male with back pain, nephrotic syndrome and

anemiapresent to the ER. Ultrasound shows normal kidney size.

His creatinine is 500. Which diagnosis best fits the scenario?

a) polycystic kidney disease

b) chronic GN

c) multiple myeloma

d) diabetic nephropathy

e) analgesic abuse

17) Which of the following are indications for dialysis in ARF?

a) severe alkalosis unresponsive to medical therapy.

b) severe acidosis unresponsive to medical therapy.

c) severe hypokalemia unresponsive to medical therapy.

d) severe hypercalcemia unresponsive to medical therapy.

e) b and d

18) Which of the following is true with respect to diabetes and kidney

disease?

a) primarily affects the tubules.

b) earliest sign is decreased GFR.

c) microalbuminuria is a late sign of DM nephropathy.

d) threshold for dialysis is same as other CRF patients.

e) BP control slows progression of DM nephropathy.

19) The various species of Campylobacter can cause diseases ranging

from acute enteritis to bacteremia. Which of the following modes of

transmission does NOT apply to Campylobacter?

a) contact with infected animals

b) contaminated food and water

c) improperly cooked poultry

d) aerolized droplets

e) person to person spread via fecal-oral route

20) Which of the following is NOT a common infectious cause of acute

diarrhea?

a) Escherichia coli

b) Shigella

c) Norwalk virus

d) Vibrio cholerae

e) Helicobacter pylori

21) Impaired coronary flow reserve is associated with each of the

following conditions EXCEPT:

a) severe aortic stenosis

b) severe systemic hypertension with left ventricular hypertrophy

c) severe mitral stenosis in the presence of atrial fibrillation

d) a totally occluded coronary artery but with excellent collateral

supply from the contralateral (i.e. opposite) coronary artery

e) an isolated 30% diameter stenosis of a coronary artery

22) Which of the following pulmonary function tests most

reliably discriminates “pure” chronic bronchitis from

emphysema?

a) total lung capacity

b) functional residual capacity

c) residual volume

d) single breath diffusing capacity

e) flow at 50% vital capacity

23) Which of the following would NOT be part of your plan for the

treatment of acute ventricular fibrillation?

a) electrical defibrillation

b) lidocaine

c) epinephrine

d) bretylium

e) manganese

24) Which of the following is not an aggravating factor of congestive

heart failure?

a) hypertension

b) thyrotoxicosis

c) alcohol

d) inactivity

e) arrhythmia

25) In the course of DKA, serum potassium levels:

a) remain unaffected

b) can appear normal but total body potassium may actually be

low

c) can appear normal but total body potassium may actually be

high

d) will naturally be corrected by insulin administration

e) none of the above

26) A2 month-old boy has a Ca of 1.80 (corrected) after an assessment

for FTT. Mother informs you she has been consistently

breastfeeding without trouble as corroborated by a visiting nurse.

What is at the top of your differential?

a) malabsorption

b) pseudohypoparathyroidism

c) mother didn’t supplement with DiVisol (Vit D supplement)

d) DiGeorge syndrome

27) Avirus that is not inactivated by mild detergents that solubilize

phospholipid membranes is:

a) poliovirus

b) variola virus

c) cowpox virus

d) vaccina virus

28) All the following are true statements about viruses EXCEPT:

a) they are obligate intracellular parasites

b) they are filterable agents

c) they are simply organized

d) they are devoid of enzymes

e) they may contain double stranded DNA

29) Aviral genome that does not replicate in the cytoplasm of the

infected cell is:

a) poliovirus

b) rabies virus

c) cytomegalovirus

d) rubella virus

e) mumps virus

30) Which of the following is true about congenital heart block in

neonatal lupus erythematosus?

a) it is associated with maternal anti-Ku autoantibodies

b) it is transient

c) the majority of patients will require a pacemaker

d) there is no increased risk of connective tissue disease in

adulthood

e) the risk of mortality is small

31) Which of the following is true about serologic testing in SLE?

a) Aapositive ANAis specific for SLE

b) ds-DNA level correlates with disease activity in SLE

c) anti-histone antibodies are seldom positive in non-drug induced

SLE

d) the majority of patients with SLE have anti-Sm antibodies

e) anti-Ro antibody is specific for SLE

32) The treatment of choice for thrombotic events in the

antiphospholipid antibody syndrome is:

a) intravenous steroids

b) high-dose oral steroids with a rapid taper

c) penicillamine

d) aspirin

e) warfarin

41) Which of the following pair of CNS lesions and corresponding

visual field defects is incorrect?

a) temporal lobe tumour – superior quadrantanopia

b) frontal lobe tumour – altitudinal field defect

c) pituitary tumour – bitemporal hemianopsia

d) occipital lobe tumour – homonomous hemianopsia

e) Multiple Sclerosis – central scotoma

42) The “triple bolus” test of pituitary function works by a rapid

succession of IV constituents as follows:

a) insulin – hypoglycemia mediated rise in GH and ACTH

GHRH – rise in LH and FSH

TRH – rise in TSH and PRL

b) CRH – rise in GH and ACTH

GHRH – rise in LH and FSH

TRH – rise in TSH and PRL

c) estrogen – rise in LH, drop in FSH and PRL

insulin – rise in GH and ACTH

TRH – TSH

d) cortrosyn – rise in GH and ACTH

GHRH – rise in LH and FSH

TRH – rise in TSH and PRL

43) A58 year-old man with a past history of a parathyroidectomy for

primary hyperparathyroidism is now in your office complaining

of headaches worse in the AM (made worse by a small MVAhe

credits to a loss of peripheral vision). You plan to:

a) send to the Emergency Department for an immediate CT head

b) check his calcium to ensure there’s no remaining parathyroid

tissue

c) check for a pheochromocytoma (which you know causes

H/As) because you are concerned he has MEN I syndrome

d) check for a homonymous hemianopia because you are

worried about a pituitary tumor

e) check for a bitemporal hemianopia because you are worried

about a pituitary tumor

44) A30 year-old patient with asthma complains of daily wheezing

and occasional waking at night with cough and chest tightness for

three weeks. His usual medication is salbutamol two puffs

tid-qid. The next step in management is:

a) add long-term theophylline

b) increase salbutamol to two puffs q4h

c) add ipratropium bromide two puffs qid

d) add beclomethasone two puffs qid

e) discontinue salbutamol and begin prednisone 50 mg od and

taper over 2 weeks

45) Which is not a feature of asbestosis?

a) Increased risk of cancer.

b) Pleural thickening and effusion.

c) Interstitial fibrosis.

d) Obstructive pattern on pulmonary function tests.

e) All of the above are features of asbestosis.

46) A63 year-old woman develops intermittent dizziness.

Examination discloses diminished corneal light reflex and mild

hearing loss in theright ear. The most likely diagnosis is:

a) cerebellopontine angle tumour

b) benign paroxysmal positional vertigo

c) lateral medullary syndrome

d) Méniére disease

e) none of the above

47) A25 year-old man is admitted with a history suggesting seizures.

Which of the following would not support this diagnosis?

a) urinary incontinence

b) the sound of voices preceding events

c) drowsiness and weakness following the event

d) rarely occur when recumbent

e) none of the above

48) Which of the following would not be expected in a right-sided

Brown-Séquard syndrome?

a) right-sided hemi-paresis

b) right-sided decreased proprioception

c) left-sided decreased sensitivity to pinprick

d) left-sided decreased vibration sense

e) none of the above

8 Sample Questions Medicine Toronto Notes 2005

33) Highly infective chronic hepatitis B is suggested by:

a) elevated liver enzymes, HBeAg+, anti-HBc IgM+

b) normal liver enzymes, HBeAg+, anti-HBc IgG+

c) normal liver enzymes, HBeAg-, anti-HBc IgG+

d) elevated liver enzymes, HBeAg+, anti-HBc IgG+

34) Which is more often associated with hospital acquired pneumonia

than community acquired pneumonia?

a) Streptococcus pneumoniae

b) Hemophilus influenza

c) Legionella

d) Chlamydia pneumoniae

e) Mycoplasma pneumoniae

35) Which of the following is least likely to contribute to myeloma?

a) hypercalcemia

b) amyloidosis

c) infiltration of the kidney by myeloma cells

d) hyperuricemia

e) intratubular light chain deposition

36) All of the following are vitamin-K dependent proteins EXCEPT:

a) protein C

b) antithrombin III

c) factor IX

d) factor II

e) factor VII

37) An 11 year-old male comes in with erythematous pustules,

inflamed nodules and cysts with some scaring distributed on the

face predominantly. Diagnosis of acne vulgaris was given.

Topical erythromycin was used for 2 weeks, several months ago,

with no response. What treatment would you prescribe now?

a) accutane immediately

b) topical tretinoin

c) topical benzoyl peroxide

d) topical antibiotic other than erythromycin

e) oral antibiotic

f) oral antibiotic and topical tretinoin

38) A40 year-old woman develops recurrent papules and pustules in

a symmetrical pattern on her cheeks, nose, chin and forehead. She

blushes easily, especially when consuming hot liquids, alcohol, or

spicy foods. The most likely diagnosis is:

a) acne vulgaris

b) perioral dermatitis

c) acne rosacea

d) seborrheic dermatitis

e) carcinoid syndrome

39) A27 year-old man is brought into the ER after a bicycling accident.

Acar door suddenly opened in front of him, of which he smashed

into and was thrown 15 feet. On examination, he is drowsy and

confused. He opens his eyes when his name is called. He mumbles

words that you understand but the sentences do not make sense.

He moves all four limbs but does not respond to any commands.

He is able to pull both hands away when pinched and squirms

when his sternum is rubbed, making no effort to stop you. What is

his Glasgow COMA Scale score?

a) 10

b) 11

c) 9

d) 8

e) 7

40) A74 year-old, right-handed man presents with a past medical

history of hypertension and dyslipidemia for 30 years. He is a

retired banker who recently has had trouble calculating his

restaurant bill. He also notices that his writing has deteriorated.

On physical exam, he has difficulty naming his fingers and is

confused with distinguishing left from right. The lesion is most

likely in which part of the brain?

a) right parietal

b) left parietal

c) left temporal

d) right temporal

e) frontal

Toronto Notes 2005 Medicine Sample Questions 9

49) Endocarditis in an I.V. drug user:

a) is equally prevalent to that of the normal population

b) is commonly located in the mitral valve

c) is typically caused by S. pneumoniae

d) is typically found on the tricuspid valve

e) is typically found on the aortic valve, producing a systolic

ejection murmur

50) A55 year-old man with a history compatible with chronic

bronchitis presents to your office with shortness of breath on exam.

In the history, all of the following would be anticipated EXCEPT:

a) A20-year history of smoking

b) worsening of symptoms with exposure to smog

c) worsening of symptoms with acute respiratory infections

d) recurrent episodes of pleurisy

e) increased incidence of chronic respiratory disease in family

members

51) Which of the following are not consistent with primary

(spontaneous) bacterial peritonitis?

a) abdominal discomfort and fever

b) ascitic fluid neutrophil count of> 250x106 cells/L

c) ascitic fluid WBC count of >500x106 cells /L

d) multiple organisms on culture and sensitivity of ascitic fluid

52) Which finding is NOT frequently found in Chronic Myelogenous

Leukemia (CML)?

a) elevated WBCs

b) elevated vitamin B12 level

c) elevated LDH

d) translocation between chromosomes 9 and 14

e) increased uric acid level

53) Schistocytes on blood film examination are UNLIKELY to be seen

in which of the following:

a) thrombotic thrombocytopenia purpura (TTP)

b) thalassemia

c) vasculitis

d) disseminated intravascular coagulation (DIC)

e) glomerulonephritis

54) All of the following can INHIBIT the absorption of ingested

non-heme iron EXCEPT:

a) alcohol

b) achlorhydria

c) phosphate (i.e. as found in milk)

d) phytates (i.e. as found in cereals)

e) antacids

55) Which of the following is true of Myasthenia Gravis?

a) in patients older than 60, thymic hyperplasia is a common

etiology

b) often associated with thyroid disease

c) antibodies that are produced against acetylcholinesterase

d) associated with small cell lung carcinoma

56) Which of the following is true with respect to proteinuria?

a) all proteinuria is secondary to glomerular disease

> 2 g/24 h = nephrotic syndrome

b) is always abnormal and indicative of serious renal disease

c) it may be normal for an individual to have <150 mg per day of

proteinuria

d) if a patient has 1.5 g of protein in 24 h they must have

tubular-interstitial disease.

57) In which of the following conditions would one not expect a

Trans-Tubular Potassium Gradient greater than 4?

a) primary hypoaldosteronism

b) acute vomiting

c) renin-secreting tumour

d) unilateral renal artery stenosis

e) Gordon syndrome

58) The most significant cause of morbidity in the elderly:

a) arthritis

b) dementia

c) heart disease

d) stroke

e) hearing impairment

59) Which of the following is NOT an age-related change?

a) impaired myocardial diastolic dysfunction

b) increased gastric acid secretion

c) decreased drug clearance

d) increased nocturnal sodium and fluid excretion

e) decreased baroreflex sensitivity

60) Regarding the elderly patient, which of the following apply?

a) vague symptoms

b) atypical presentations

c) loss of function

d) polypharmacy

e) all of the above

61) Which of the following is not associated with thyroid disease?

a) dermatitis herpetiformis

b) urticaria

c) porphyria cutanea tarda

d) vitiligo

e) alopecia areata

62) All of the following are treatments for non-scarring alopecia except:

a) spironolactone

b) minoxidil

c) hair transplantation

d) intralesional triamcinalone

e) finasteride

63) A70 year-old woman presents with acute knee arthritis.

Radiographs show meniscal calcification (chondrocalcinosis).

Analysis of the synovial fluid reveals weakly positive

birefringent rhomboid-shaped crystals. The crystals are

most likely:

a) monosodium urate

b) calcium hydroxyapatite

c) cholesterol

d) calcium pyrophosphate dihydrate

e) dicalcium phosphate dihydrate (Brushite)

64) Entamoeba histolytica is transmitted to humans by:

a) ingestion of infective eggs

b) ingestion of cysts

c) ingestion of animal tissue that contains the larva

d) penetration of the skin by infective larva

e) ingestion of adult form

65) Strongyloides spp. is transmitted to humans by:

a) ingestion of infective eggs

b) ingestion of cysts

c) ingestion of animal tissue that contains the larva

d) penetration of the skin by infective larva

e) ingestion of adult form

66) The biosynthesis of fungal ergosterol is inhibited by:

a) amphotericin B

b) griseofulvin

c) flucytosine

d) nystatin

e) ketoconazole

67) A55 year-old woman with asthma is on systemic steroids for one

year. She develops a recent right-sided pleural effusion. She feels

unwell and tires easily. Aspiration reveals a turbid fluid, a high

lymphocyte count, high LDH, low glucose, and a pH of 7.4. The

most compatible diagnosis is:

a) pulmonary embolism

b) empyema

c) tuberculosis

d) subphrenic abscess

e) pancreatitis

74) Nitroglycerin administered sublingually may contribute to the

relief of myocardial ischemic pain by each of the following

mechanisms EXCEPT:

a) coronary vasodilation

b) decreased venous pooling resulting in increased cardiac

preload

c) reduced systemic vascular resistance

d) reduced ventricular volume

75) At what CD4 count are HIV patients at increased risk of

developing PCP?

a) CD4 count >500x106

b) CD4 count 200-499x106

c) CD4 count <200x106

d) CD4 count < 500x106

e) none of the above

76) An 80 year-old female with a 25% reduction in her GFR requires

which of the following adjustments for a drug that is

predominantly cleared by the kidneys?

1. 25% decrease in individual doses

2. 25% increase in dosing intervals

3. increase overall fluid intake

4. decrease length of therapy by 25%

a) all of the above

b) 1 and 3

c) 2 and 4

d) 2,3, and 4

e) 1 and 2

77) Consequences of immobility include which ONE of the following:

a) diarrhea

b) maintenance of muscle mass

c) urinary retention

d) pneumonia

e) hastened wound healing

78) Which of the following is FALSE in terms of falls in the elderly?

a) they are the most common cause of mortality due to injury

b) environment plays a significant role

c) fractures most commonly involve the humerus

d) age-related sensory changes make the elderly more

susceptible

e) fear of falling contributes to self-protection immobility

10 Sample Questions Medicine Toronto Notes 2005

68) Diagnosis of acute symptomatic pulmonary embolism can be

excluded when which of the following is normal?

a) chest x-ray

b) ventilation-perfusion lung scan

c) bilateral leg venograms

d) PaO2 and A-a O2 gradient

e) CT scan of the pulmonary arteries

69) In acute pyelonephritis, which of the following is most commonly

associated with bacteremic spread from a distant focus?

a) Escherichia coli

b) Proteus sp.

c) Staphylococcus aureus

d) Serratia sp.

e) Enterococcus sp.

70) Apatient presents with a decreased level of consciousness and

visual difficulties. Bloodwork reveals an anion gap of 22 and an

osmolar gap of 24. Which of the following is most likely

responsible?

a) ethanol

b) salicylates

c) renal tubular acidosis type I

d) methanol

e) diabetic ketoacidosis

71) In the treatment of Type I Diabetes, which of the following is true?

a) sulfonylureas are useful as an adjunctive therapy to insulin

b) most patients are adequately controlled with one type of

insulin (non-mixed) only

c) once diagnosed with Type I, patients must immediately be

assessed for retinopathy

d) during periods of illness or infection, patients may require

additional insulin

e) the most common initial presentation is visual disturbance

72) Which of the following is associated with thyroid disease?

a) neurofibromatosis

b) vitiligo

c) erythema nodosum

d) pemphigus vulgaris

e) icthyosis vulgaris

73) Each of the following is a correct statement about COPD EXCEPT:

a) the type of emphysema associated with smoking is usually

centriacinar

b) clubbing is not a clinical feature

c) long-term oral steroids should be avoided

d) smoking cessation does not lead to improvement of

pulmonary function

e) the aim of supplemental O2 therapy is to provide relief of

shortness of breath

Toronto Notes 2005 Obstetrics and Gynecology Sample Questions 11

Obstetrics and Gynecology

1) A26 year-old primigravida presents at 40 weeks in active labour

with contractions every 2 minutes. She is diagnosed as having a

transverse lie with the back up. Which of the following would be

the most appropriate next step?

a) start isoxuprine (Vasodilan)

b) perform an external version

c) prepare for an immediate C-section

d) rupture membranes and then perform an internal version

e) none of the above

2) A25 year-old primigravida at 34 weeks gestation is thought to be

small for dates by her physician and is sent for a sonographic

evaluation. The ultrasound shows the biparietal diameter to be

appropriate for 34 weeks gestation. The abdominal circumference

is appropriate for 30 weeks gestation. The head:abdominal

circumference ratio is < 1. The estimated fetal weight is <10th

percentile for 34 weeks gestation. The amniotic fluid is decreased.

Which of the following is the most likely diagnosis?

a) symmetrical IUGR

b) asymmetrical IUGR

c) congenital anomaly

d) congenital infection

e) unknown gestational age

3) A32 year-old G5P4 presents with an 8 week history of amenorrhea

andsuggestive symptoms of pregnancy. Physical examination

reveals an irregular, enlarged uterus of 16 weeks size. Ultrasound

confirms thepresence of an 8 week viable pregnancy and a

multiple fibroid uterus. The correct management for this patient is:

a) termination of the pregnancy with elective myomectomy

2 months later

b) termination of the pregnancy with concomitant myomectomy

c) prudent observation with elective C-section at term

d) prudent observation anticipating probable vaginal delivery

e) myomectomy and follow pregnancy in usual way

4) An infant is born. At one minute the heart rate is 120 per minute,

respiratory effort is a good strong cry, muscle tone is active, reflex

irritability is absent, colour is pink with blue extremities. What is

the one minute Apgar score?

a) 5

b) 6

c) 7

d) 8

e) 9

5) Which of the following is the easiest and most reliable way of

detecting a retained succenturiate placental lobe?

a) palpation of the uterus

b) inspection of the maternal side of the placenta

c) visualization of the cervix on speculum exam

d) inspection of the fetal side of the placenta

e) pelvic ultrasound

6) Choose the correct statement regarding operative obstetrics:

a) a first degree tear involves skin and vaginal mucosa as well as

underlying fascia and muscle

b) delivery aided by vacuum extraction is typically of shorter

duration than if forceps are used

c) one absolute prerequisite for the use of forceps is that the

baby’s presenting part must be below the ischial spines

d) a third degree tear extends through the anal sphincter

e) none of the above

7) A36 year-old female presents with a mucopurulent vaginal

discharge. Gram stain of a cervical swab shows gram negative

diploccocci. Which of the following is correct?

a) if untreated this condition will likely resolve spontaneously

b) the most likely diagnosis is Strep group B

c) appropriate treatment is Metronidazole vaginal cream

d) appropriate treatment is a single I.M. dose of 250 mg

of Ceftriaxone

e) Public Health will most likely need to be notified

8) Aone year-old female presents with vaginal bleeding. Vaginal

inspection reveals the presence of a multicystic grape-like lesion.

The most likely diagnosis is?

a) sexual abuse

b) DES syndrome

c) sarcoma botyroides

d) clear cell adenocarcinoma

e) exposure to exogenous estrogen

9) A24 year-old G1P0 is seen in the Emergency Department. Her LMP

was 8 weeks ago. She is experiencing lower abdominal cramping

and heavy vaginal bleeding with clots. Examination reveals a soft

abdomen with mild lower abdominal tenderness. On pelvic exam,

the vagina is filled with blood and clots. The cervical os is opened

and tissue is protruding. The uterus is enlarged to a 6 week size.

Which of the following is the most likely diagnosis?

a) ectopic pregnancy

b) threatened abortion

c) degenerating fibroid

d) placenta previa

e) incomplete abortion

10) A24 year-old female G1P0 is admitted to hospital at 34 weeks

gestation. The patient describes passing a large volume of clear fluid

per vagina for 36 hours. Examination reveals the following: temp

38.5C, no uterine contractions, vague lower abdominal tenderness,

fetal heart rate 185 bpm. Lab investigations reveal: Hb 120, WBC

19.0, + nitrazine test of vaginal fluid, + ferning of vaginal fluid.

Which of the following is the most likely diagnosis?

a) premature rupture of membranes

b) premature rupture of membranes and chorioamnionitis

c) premature labour

d) acute pyelonephritis

e) acute vaginitis of pregnancy

11) A24 year-old female presents with abdominal pain. ß-HCG is

negative. Pelvic ultrasound shows a 5 cm right ovarian cyst. You

would:

a) perform immediate laporotomy

b) perform immediate laproscopy

c) aspirate the cyst under ultrasonographic guidance

d) order a CBC and a CA125

e) expectant management with repeat ultrasound in 8 weeks

12) The pathology report following a therapeutic abortion shows

Arias-Stella phenomenon (decidualization). The most appropriate

next step in the management of this patient would be?

a) repeat a pregnancy test

b) laporoscopy

c) reassure the patient

d) prescribe oral contraception

e) repeat the D&C

13) In a patient whose blood type is O-Rh negative and whose husband

has O-Rh positive blood, Rh immune globulin (Rhogam) should be

given after all the following EXCEPT:

a) artificial rupture of membranes

b) full term delivery of a baby with O-Rh positive blood type

c) amniocentesis

d) spontaneous abortion

e) therapeutic abortion

14) Risk factors for shoulder dystocia include all the following EXCEPT:

a) maternal obesity

b) macrosomia

c) maternal diabetes

d) prolonged second stage of labour

e) outlet forceps delivery

25) A55 year-old woman comes to the office to discuss menopause.

You tell her all of the following are a part of this stage EXCEPT:

a) hot flushes

b) vaginal dryness

c) mood swings

d) insomnia

e) decreased libido

26) All of the following features have been described in women with

androgen insensitivity syndrome EXCEPT:

a) mammary aplasia

b) female phenotype

c) absence of wolffian duct structures

d) short vagina

e) absence of mullerian duct structures

27) The commonest indication for amniocentesis is:

a) chromosome anomaly of the mother

b) teenage pregnancy

c) advanced maternal age

d) family history of Down syndrome

e) family history of cystic fibrosis

28) On colposcopy, the transformation zone:

a) contains columnar epithelium

b) is the area between original squamous epithelium and columnar

epithelium

c) contains metaplastic epithelium

d) contains atypical blood vessels

29) Complications of cesarean section include each of the

following EXCEPT:

a) hemorrhage

b) infection

c) laceration of the fetus

d) subsequent rupture of the uterine scar

e) inversion of the uterus

30) Regarding laparoscopy:

a) it is contraindicated in patients who are menstruating.

b) patients with intestinal obstruction are still able to undergo

laparoscopy

c) the most common indication for therapeutic laparoscopy is tubal

sterilization

d) all of the above are incorrect

31) Causes of secondary amenorrhea include:

a) Turner syndrome

b) anorexia

c) androgen insensitivity syndrome

d) gonadal dysgenesis

e) imperforate hymen

32) Risk factors for cervical carcinoma includes all of the

following EXCEPT:

a) human papilloma virus type 16 and 18

b) early age at first intercourse

c) smoking

d) nulliparity

e) multiple sexual partners

33) A35 year-old G0P0 has severe pain during menses which radiates

into the anal region. On bimanual exam, she has a small, retroverted

uterus and tender nodules can be felt on palpation of the uterosacral

ligaments. Which of the following conditions does she most likely

have:

a) chronic PID

b) adenomyosis

c) fibroids

d) endometriosis

e) uterine carcinoma

34) Which of the following statements is correct?

a) cervical carcinoma is the most common gynecological

malignancy in Canada

b) lichen sclerosis is treated with estrogen cream

c) colposcopy is required after a finding of mild cervical

dysplasia on a routine Pap smear

d) follicular cysts typically regress with the following cycle

e) leiomyomata are malignant tumours

12 Sample Questions Obstetrics and Gynecology Toronto Notes 2005

15) Fetal exposure to lithium in the first trimester has been strongly

associated with:

a) alopecia

b) cleft lip and palate

c) congenital heart defects

e) limb defects

d) mental retardation

16) The commonest type of bleeding encountered with uterine

leiomyomata is:

a) post-coital spotting

b) mid-cycle bleeding

c) hypermenorrhea

d) oligomenorrhea

e) post-coital staining

17) Which of the following statements about estrogen therapy in

postmenopausal women is correct?

a) it is a major risk factor for breast cancer

b) it decreases hepatic triglyceride production

c) it produces hypertension

d) it protects against vertebral compression fractures

e) it directly stimulates the proliferation of vascular endothelium

18) Which event is matched with the correct timing?

a) maternal serum screening test: 20-22 weeks

b) post-partum visit: 6 weeks

c) chorionic villus sampling: 4-6 weeks

d) oral glucose challenge test: 14-16 weeks

e) Rhogam to Rh negative women: 12-14 weeks

19) Routine blood work during the first antenatal visit include all of

the following EXCEPT:

a) CBC

b) HIV

c) rubella titre

d) blood group and type

e) Rh antibodies

20) All of the following are cardiovascular adaptations to pregnancy

EXCEPT:

a) cardiac output increases 33-45%

b) stroke volume increases 10-30%

c) heart rate decreases 12-18 beats per minute

d) systolic blood pressure decreases 4-5 mmHg

e) diastolic blood pressure decreases 8-15 mmHg

21) Acouple are unable to conceive after 1 year. Of the following

statistics regarding infertility, which are correct:

a) 10-15% of couples are infertile

b) only 40% of couples achieve pregnancy within 6 months of

trying

c) only 20% achieve pregnancy within 1 year of trying

d) male factors are responsible for over half of infertility causes

e) infertility is labelled after failure to conceive within 2 years of

trying

22) Vaginal discharge which is fishy in odor and associated with

>20% clue cells on microscopy will not be associated with which of

the following:

a) be due to gardnerella vaginalis overgrowth

b) can be treated with metronidazole or clindamycin

c) with the addition of KOH may see hyphae or spores

d) must be treated in all pregnant women, including

asymptomatic

e) is rarely associated with inflamed or itchy vulva

23) In the treatment of an ectopic pregnancy with methotrexate all of

the following are true EXCEPT:

a) the pregnancy must not have ruptured

b) it must be less than 5cm in size

c) patient should be clinically stable

d) compliance and f/u are ensured

e) no fetal heart activity has been noted

24) Which of the following is associated with breech presentations?

a) fetal heart sounds heard best in the upper abdomen

b) brachial plexus injury

c) prolapse of the umbilical cord

d) increased fetal mortality

e) all of the above

Toronto Notes 2005 Obstetrics and Gynecology Sample Questions 13

35) If a mother nurses her child, the time of reappearance of her

menses is uncertain but usually occurs after her delivery by:

a) 1-2 months

b) 2-3 months

c) 3-4 months

d) 4-5 months

e) > 5 months

36) Which of the following statements regarding malignant cervical

lesions is true?

a) 95% are squamous cell carcinoma

b) CA-125 levels to monitor treatment effectiveness are indicated

c) the majority of lesions arise outside the transformation zone of

the cervix

d) they are not associated with HPV infection

e) treatment for stage 4 disease radical hysterectomy with

chemotherapy

37) In endometriosis, the most common location for disease is:

a) broad ligament

b) cul-de-sac

c) ovaries

d) appendix

e) uterosacral ligament

38) All of the following increase the risk of developing ovarian cancer

EXCEPT:

a) family history

b) BCP

c) nulliparity

d) late menopause

e) caucasian

39) In the first 100 days of pregnancy, human chorionic gonadotropin

titre doubles:

a) every day

b) every week

c) every 2 days

d) every month

e) every 2 weeks

40) Ayoung female enters your office wanting to start oral

contraceptive pills. You remind yourself of the reasons she may

not start the pill which are:

a) impaired liver function

b) undiagnosed abnormal uterine bleeding

c) congenital hyperlipidemia

d) past history thrombophlebitis

e) all of the above.

41) Which of the following statements regarding endometriosis is

incorrect?

a) theories proposed to explain the histogenesis of endometriosis

include retrograde menstruation and coelemic metaplasia

b) CA-125 is often useful in making the diagnosis of

endometriosis

c) symptoms of endometriosis may include urinary frequency

and diarrhea

d) the differential diagnosis of endometriomas include

hemorrhagic

corpus luteum cysts as well as neoplasms

e) the risk of endometriosis is several fold greater if there is a

first degree relative with this condition

42) A55 year-old female presents with a "period-like" vaginal

bleeding for 5 days that stopped spontaneously. Your approach

to the problem would be to:

a) reassure the patient and manage expectantly

b) administer provera 10 mg od for 15 days

c) administer cyclic estrogen plus progesterone

d) obtain endometrial tissue

e) perform a pap smear plus endocervical curretage

43) The appropriate next step in a woman found to have 1 abnormal

Pap smear showing mild dysplasia (CIN1) is:

a) repeat in 1 year's time

b) repeat in 4-6 months time with colposcopy if abnormal

c) send patient directly for colposcopy and LEEP excision

d) send for colposcopy after 1 abnormal Pap

e) repeat Pap immediately and treat for HPV infection

44) The commonest cause of disseminated intravascular

coagulation during pregnancy is:

a) amniotic fluid embolism

b) abruptio placentae

c) placenta previa

d) missed abortion

e) chorioamnionitis

45) Contraindications to vaginal delivery include each of the

following EXCEPT:

a) previous classical cesarean section

b) squamous cell carcinoma in situ of the cervix

c) total placenta previa

d) transverse lie with ruptured membranes

e) primigravida breech with cephalo-pelvic disproportion

46) In a differential diagnosis of third trimester bleed, which of the

following should not be included:

a) placenta previa

b) vasa previa

c) abruptio placentae

d) threatened abortion

e) marginal sinus hemorrhage

47) A41 year-old woman presents with obesity, hirsutism and

oligomenorrhea, which of the following tests will give the least

useful information?

a) FSH

b) LH

c) testosterone

d) TSH

e) urine free cortisol

48) With respect to androgen insensitivity syndrome, which of the

following is true?

a) genotype is XY

b) breast development and uterus are absent

c) serum testosterone is below normal male range

d) secondary sex characteristics are male

e) the condition is autosomal recessive

49) The new selective estrogen receptor modulators have all of the

following benefits EXCEPT:

a) no agonistic effect on breasts

b) provides estrogenic benefit on bone

c) mimics estrogenic effect on cardiovascular status

d) no agonistic effect on uterine tissue

e) provides relief of hot flushes associated with menopause

50) Regarding birth control, which of the following is false?

a) it is contraindicated in women over age 35 who are smokers

b) its efficacy may be decreased when taking antibiotics

c) reduces the risk of ovarian carcinoma

d) is associated with only a 10% failure rate

e) can be associated with breast changes and weight gain.

7) Which of the following investigations is most helpful in the

assessment of a child presenting with an acute asthma attack?

a) white cell count and differential

b) arterial blood gases

c) chest x-ray

d) pulmonary function tests

e) O2 sat. monitoring

8) A6 year-old boy is brought to emergency with a 5 day history of

fever, cough, and poor appetite, but no vomiting. On exam, he

appears unwell, is febrile, and has crusty nasal discharge and a

wet-sounding cough. Chest x-ray reveals a pulmonary infiltrate in

the right middle lobe. He is admitted to hospital for pneumonia.

What is the best management for this child?

a) observe for 24 hours with IV fluids only

b) observe for 24 hours with IV fluids and acetaminophen

c) give oral amoxicillin and acetaminophen

d) give IV ampicillin and oral acetaminophen

e) give oral erythromycin and acetaminophen

9) Side effects of methylphenidate (Ritalin) therapy for hyperactivity

include each of the following EXCEPT:

a) anorexia

b) insomnia

c) tachycardia

d) dependence (addiction)

e) growth suppression

10) In comparing breast milk and formula, which of the following

statements is NOT correct:

a) breast milk has a higher percentage of protein

b) breast milk has whey:casein ratio of 60:40

c) breast milk contains leukocytes, complement and lysozymes

d) breast milk has a lower concentration of iron than iron

fortified formulae

e) breast milk has an optimal calcium:phosphorus ratio of 2:1

11) Most umbilical hernias in children:

a) need strapping

b) resolve spontaneously

c) require elective surgery

d) require immediate surgery

e) are associated with a higher incidence of inguinal hernias

12) Anewborn male spits up his first feeding and develops bilious

emesis with subsequent feedings. On physical exam he appears

ill, has a scaphoid abdomen and absent bowel sounds.

Abdominal x-ray shows air in the proximal small bowel, but a

paucity of air in the distal digestive tract. The most likely cause

for this infant's vomiting and clinical finding is:

a) antral web

b) choldochal cyst

c) Hirschsprung disease

d) tracheoesophageal fistula

e) volvulus

13) Breastfeeding is contraindicated if the mother has:

a) acute EBV infection

b) acute HepA infection

c) chronic HepB infection

d) asymptomatic HIV infection

e) none of the above

14) A10 month-old child develops low-grade fever and sunburn-like

erythema over much of the body, but most prominently

in the intertriginous areas. Within 36 hours, sheet-like

desquamation is noted in the flexures and around the mouth.

The mucous membranes are spared. Which of the following is

the most likely diagnosis:

a) Kawasaki disease

b) staphylococcal scalded skin syndrome

c) Stevens-Johnson syndrome

d) toxic epidermal necrolysis

e) toxic shock syndrome

14 Sample Questions Pediatrics Toronto Notes 2005

Pediatrics

1) A6 month-old infant presents in the winter with fever, cough,

wheezing, tachypnea and decreased appetite. Achest

radiograph shows hyperaeration and streaky perihilar infiltrates

bilaterally. You diagnose bronchopneumonia. Which organism

would most likely be causing this child's infection?

a) Chlamydia pneumoniae

b) Mycoplasma pneumoniae

c) Streptococcus pneumoniae

d) Haemophilus influenzae

e) Respiratory syncytial virus

The following case pertains to questions 2 and 3:

A12 month-old girl is brought to the emergency department for

the second time in 2 days for vomiting and passage of 8 to 10

watery stools per day.

2) Of the following, which provides the best estimate of the

patient’s volume deficit:

a) weight change since the beginning of the illness

b) hydration of mucous membranes, skin turgor, and

level of consciousness

c) pulse, blood pressure, and peripheral capillary filling time

d) serum electrolytes

e) serum urea nitrogen and creatinine levels

3) The patient has lost 0.6 kg. She is moderately lethargic and has

dry mucous membranes and reduced skin turgor. Blood pressure

is 80/40 mm Hg, and pulse is 120 per minute; capillary refill is

reasonably brisk. Lab studies reveal: sodium 131, potassium 4.8,

chloride 101, bicarbonate 16 mEq/L, urea nitrogen 24 mg/dL and

creatinine 0.6 mg/dL. The best strategy for managing this child is

to:

a) hospitalize for administration of IV fluid therapy

b) administer an oral rehydrating solution while the

child is under medical supervision for 4-6 hours

c) instruct the parents about oral rehydration at home

d) hospitalize after giving 20 mL/kg of 0.9% saline IV

e) instruct parents on use of soy formula

4) Which of the following statements about Attention Deficit and

Hyperactivity Disorders (ADHD) is INCORRECT?

a) ADHD is more commonly seen in boys than girls

b) deafness and seizure disorders must be excluded before

the diagnosis of ADHD is made

c) three characteristic features of ADHD are inattention,

hyperactivity, and mental retardation

d) ritalin is effective in the treatment of about 70% of cases of

ADHD

e) side effects of Ritalin include poor growth, insomnia, and tics

5) A6 year-old child is brought into the Emergency with his father

because of a fractured arm. Although his father states that the

child fell off his bike, the fracture does not seem consistent with

this story. On inspection, you discover that the child seems to

have a number of old bruises on his back and on his legs, some

of which seem reminiscent of finger markings. Which of the

following steps is not appropriate in this circumstance?

a) let the child go home with his father since you cannot really

prove anything

b) document the injuries that you saw with possible

photographs

c) call the CAS and inform them

d) try to interview the child alone

e) do additional investigations (e.g. skeletal survey)

6) A14 year-old girl has refused to go to school 3 times in the past

4 months. She says, "My tummy hurts," but she cannot point to

where it bothers her. Her appetite is good, her bowel movements

are normal, and she is sleeping well at night. She is a

healthy-looking girl with no abnormal findings on physical exam.

What is the most likely diagnosis?

a) a benign abdominal mass compressing her duodenum

b) diaphragmatic hernia

c) somatoform disorder

d) gastroesophageal reflux disease

e) early peptic ulcer disease

Toronto Notes 2005 Pediatrics Sample Questions 15

15) By one year of age, a child is able to have all of the following

foods except:

a) breast milk

b) peeled fruits

c) cheese

d) peanuts

e) eggs

16) Central cyanosis in the newborn infant is most often caused by:

a) congenital heart disease

b) lung disease

c) central nervous system disease

d) methemoglobinemia

e) hypoglycemia

17) In a premature infant who is suspected of having necrotizing

enterocolitis (NEC), each of the following is correct EXCEPT:

a) Septicemia is associated with an increased risk of NEC

b) NEC is thought to be caused by systemic hypertension

c) the finding of air in the portal vein indicates severe illness

d) respiratory distress increases the risk of NEC

e) apgar scores inversely correlate with the risk of NEC

18) Which of the following is the most significant risk factor for the

development of childhood asthma:

a) family social background

b) parental asthma

c) stress in the family

d) parental smoking

e) presence of pets in the house

19) Which of the following statements about sickle cell disease is true?

a) all patients with sickle cell disease have a homozygous HbSS

genotype

b) sickle cell disease causes a severe chronic anemia that is not r

outinely transfusion dependent

c) patients have increased susceptibility to infection by

nonencapsulated organisms

d) patients usually present with sickle cell crises within one month

of age

e) splenic dysfunction usually does not occur until the child enters

his/her teens

20) Which of the following is false about physiological jaundice?

a) it rarely presents before the age of 24 hours

b) it may cause kernicterus

c) in premature infants it may persist for 3-4 weeks

d) it is mainly due to temporarily impaired hepatic clearance of

bilirubin

e) it does not always require treatment with phototherapy

21) Which of the following statements is true?

a) cancer is the second most common cause of death in children

b) Hodgkin's lymphoma is the most common childhood cancer

c) hyperploidy in leukemic cells is a poor prognostic indicator

d) Wilm's tumor is rarely associated with other congenital

abnormalities

e) neuroblastomas usually occur in late adolescence

22) A5 year-old girl with hypogammaglobulinemia and absent

immunoglobulin A(IgA) receives infusions of immune

globulin monthly. Shortly after her most recent infusion

began, she developed hypotension, wheezing, and several

urticarial lesions. Which of the following best explains her

reaction?

a) anaphylactic reaction due to IgE anti-IgA antibodies

b) Gram-negative sepsis due to contaminated immune globulin

c) idiosyncratic reaction due to rapid infusion of immune

globulin

d) serum sickness reaction from foreign serum in the immune

globulin

e) transfusion reaction due to ABO incompatibility

23) When prophylactic antibiotic therapy is used for tooth

extraction in a patient with a ventricular septal defect:

a) therapy is started 24 hours prior to the procedure

b) a throat swab should be taken prior to the procedure

c) a second generation cephalosporin (cefuroxime) is the

therapy of choice

d) therapy is given for one week following the extraction

e) erythromycin is the drug of choice for those allergic to

penicillin

24) Which of the following statements about stuttering in a

4 year-old is incorrect:

a) it is characterized by intermittent difficulty in producing a

smooth flow of speech

b) it is more than 3 times more frequent in girls than in boys

c) it is exacerbated by anxiety

d) more than 30% of children who stutter recover

spontaneously

e) none of the above

25) Awoman who is positive for hepatitis B surface antigen

(HBsAg), but negative for hepatitis B antigen (HBeAg),

delivers at term. What would be the best management for

this woman's infant?

a) administer gamma globulin intramuscularly immediately

and at 1 month of age

b) administer hepatitis B (HB) vaccine immediately and at 1

month and 6 months of age

c) administer hepatitis B immune globulin (HBIG) if cord

blood is positive for HbsAg

d) administer HBIG and HB vaccine immediately, and HB

vaccine again at 1 month and 6 months of age

e) advise mother that breastfeeding is contraindicated

26) Regarding sexual abuse of children, each of the following

statements is true EXCEPT:

a) no genital injury is found in the majority of patients

b) father-daughter incest is more common than brother-sister

c) most assailants are unknown to the victim

d) laboratory findings usually show no presence of sperm

e) half of the abused children come from single parent

families

27) The viral infection most likely to cause CNS involvement and

focal neurological findings is :

a) coxsackievirus

b) herpes simplex

c) enterovirus

d) rabies

e) rhinovirus

28) Which of the following predisposes infants to chronic otitis media?

a) bottle-feeding in upright position

b) abnormal tympanic membrane formation

c) environmental factors such as daycare and passive smoking

d) allergies

e) none of the above

29) A13 month-old infant boy has chronic diarrhea, poor appetite,

irritability and growth failure. He had been well previously,

developing normally until diarrhea began at 3 months of age.

Findings include weight loss less than 5th percentile and length

at 25th percentile, cachectic appearance, wasted extremities and

protuberant abdomen. His labs are: albumin 23, protein 40.

Stool positive for reducing sugars and negative for enteric

pathogens and ova and parasites. These findings are most

consistent with:

a) celiac disease

b) cow milk allergy

c) Crohn disease

d) Cystic Fibrosis

e) none of the above

38) An 8 year-old boy has had paroxysmal abdominal pain since his

parents separated 6 months ago. Which of the following

symptoms would support and organic basis for his disease?

a) headaches accompanies the pain

b) pain located in the periumbilical region

c) pain awakens child at night

d) symptoms last < 1hr

e) none of the above

39) The most common cause of chronic diarrhea in a

6 month - 36 month old child is:

a) toddler’s diarrhea

b) disaccharidase deficiency

c) enteric infection

d) malabsorption

e) protein intolerance

40) Which of the following is NOT required for a diagnosis of

juvenile rheumatoid arthritis?

a) arthritis in at least one joint

b) arthritis lasting for at least 6 weeks

c) positive rheumatoid factor

d) onset before the age of 16

e) other causes of arthritis excluded

41) Achild presents with bilateral shin pain. Which of the following

suggests that this is NOT growing pains?

a) pain is poorly localized

b) pain awakens the child at night

c) no fever or rash

d) pain abates with reassurance and massage

e) child may limp in the morning from stiffness

42) Which of the following statements about precocious puberty is

true?

a) the cause is never found in most cases

b) the children are always shorter than average

c) the incidence is the same for males and females

d) ketoconazole is the treatment of choice for females

e) all girls with breast development before age 10 require

karyotyping for Turner syndrome

43) All of the following groups are contraindicated to receive the oral

polio vaccine EXCEPT:

a) pregnant women

b) asymptomatic HIV-positive children

c) household contacts of immunodeficient children

d) unimmunized adults at future risk of being exposed to the

polio virus

e) children with agammaglobulinemia

44) Which of the following steps is NOT indicated in the

management of croup?

a) keeping the child calm

b) hydration

c) antipyretics

d) antibiotics

e) humidified oxygen

45) Which of the following pairs shows the CORRECT stage of

normal development?

a) 6 months - pulls to stand

b) 12 months - pincer grasp

c) 18 months - handedness

d) 2 years - draws a cross

e) 3 years - tells a story

46) Which of the following is NOT characteristic of a functional

murmur?

a) pansystolic murmur

b) murmur varies with position

c) variably split S2

d) murmur becomes louder with fever

e) no extra clicks

16 Sample Questions Pediatrics Toronto Notes 2005

30) All infants less than 3 months of age who have fever and no

localizing signs should have all of the following evaluations

EXCEPT:

a) blood cultures

b) stool cultures

c) chest x-ray

d) CBC (with differential)

e) urine culture

31) All the following statements regarding transient tachypnea of the

newborn (TTN) is true EXCEPT:

a) infants born by C-section are at increased risk for developing

TTN

b) residual pulmonary function disability is common among

infants who have TTN

c) the incidence of TTN is higher than Respiratory Distress

Syndrome (RDS) among term infants

d) TTN shows marked improvement with 12-24 hrs

e) chest x-ray findings consisntent with TTN include increased

pulmonary vasculature and fluid in the fissures

32) Common signs of heart failure in children include all of the

following EXCEPT:

a) decreased exercise tolerance

b) fatigue

c) failure to thrive

d) frequent URTIs

e) peripheral edema

33) Antibiotic prophylaxis against infective endocarditis is required for

all of the following EXCEPT:

a) rheumatic valve lesions

b) prosthetic heart valves

c) isolated secundum ASD

d) pacemaker leads

e) PDA

34) Causes of microcytic anemia include all of the following EXCEPT:

a) excessive cow's milk intake

b) iron deficiency

c) folic acid deficiency

d) thalassemias

e) lead poisoning

35) The latest time after the onset of GABHS pharyngitis that initiation

of penicillin therapy can be expected to prevent acute rheumatic

fever is:

a) 24 hrs

b) 48 hrs

c) 96 hrs

d) 9 days

e) 2 weeks

36) Fetal tricuspid valve abnormalities are associated with

maternal use of which of the following:

a) lithium

b) warfarin

c) chloramphenicol

d) estrogen

e) heroin

37) Which of the following is most likely to provide the basis for

making the correct diagnosis in an infant or child who has failure to

thrive (FTT) ?

a) blood chemistries

b) cultures

c) history and physical

d) radiograph studies

e) none of the above

Toronto Notes 2005 Pediatrics Sample Questions 17

47) Which of the following is the most common congenital heart

lesion in children?

a) ASD

b) VSD

c) PDA

d) Tetralogy of Fallot

e) coarctation of the aorta

48) Choose the INCORRECT statement about breastfeeding:

a) for healthy term babies, breastfeeding is recommended over

formula feeding

b) breastmilk contains more amino acids than cow’s milk

c) exclusively breastfed babies should be supplemented with

iron after 6 months of age

d) assessment of adequate intake can be done by counting

the number of wet diapers in a day

e) an HIV-positive mother should not breastfeed

49) All of the following are characteristics of a child with Pervasive

Developmental Disorder (PDD) EXCEPT:

a) abnormal speech pattern

b) stereotypic behaviours, such as head-banging or hand-flapping

c) abnormal cognitive function

d) consuming interest in one topic or activity

e) tendency to reciprocate in peer interactions

50) All of the following are appropriate in the initial assessment of a

child with failure to thrive EXCEPT:

a) diet history

b) social history

c) measurement of height, weight and head circumference

d) growth hormone levels

e) bone age x-ray

51) Which of the following statements about a child with short stature is

CORRECT?

a) the bone age is delayed in a child with constitutional growth

delay

b) height crosses a major percentile line on the child’s growth chart

c) weight is more affected than height in a child with an endocrine

deficiency

d) karyotyping is part of the routine investigation of all children

with short stature

e) growth hormone replacement is helpful in a child with familial

growth delay

52) An 8 year-old boy who has been continent of urine between the

ages of 4 and 5 years comes in because of recent onset of

enuresis. Which of the following statements is false?

a) it may be precipitated by a family crisis

b) it does not require a specific treatment

c) it can lead to poor self esteem

e) it responds best to individual psychotherapy

e) it can be secondary to diseases such as diabetes mellitus

53) Which of the following is NOT correct about the pertussis

vaccine:

a) delaying the primary series until the age of 1 year will not

reduce the risk of a seizure

b) the commonly quoted risk of 1:330,000 of brain damage has

no basis in fact

c) it can cause febrile seizures

d) it may cause infantile spasms or sudden infant death

syndrome

e) the risk of other forms of neurologic illness eg. transverse

myelitis is extremely small

54) The most common cause of apnea in infants less than

6 months is:

a) encephalitis

b) seizure disorder

c) cardiac arrhythmia

d) milk allergy

e) gastroesophageal reflux

55) A7 day-old breastfed infant born at term has had decreased

appetite, irritability, and vomiting for 24 hours. On physical

examination, the infant appears listless. Respiratory rate is

40/min; heart rate, 160/min; and blood pressure, 68/38 mm Hg.

he skin and sclerae are icteric but no other abnormalities are

noted. Laboratory studies reveal: hemoglobin 120 g/L; total

bilir bin, 270 umol/L; and direct bilirubin, 135 umol/L.

Urinalysis is negative for reducing substances. Which of the

following is the most likely diagnosis:

a) bacterial sepsis

b) blood group incompatibility

c) breast milk jaundice

d) hypothyroidism

e) intrauterine infection

56) Each of the following statements about rotavirus gastroenteritis

in children is correct except:

a) it usually occurs in the winter months

b) it is self-limited

c) it is often associated with respiratory symptoms

d) it is often accompanied by fever and vomiting

e) it is often associated with grossly bloody diarrhea

57) The most common cause of deterioration of diabetic control

in an 8 year-old boy is:

a) change in diet

b) lack of compliance

c) emotional stress

d) acute infection

e) change in exercise level

58) Achild born at 32-weeks gestation is brought to your office at

2 months of age for a routine check-up. The child has an

upper respiratory tract infection but is afebrile. What decision

would you make regarding the child's first vaccination?

a) wait until the child is two months older to administer the

first vaccination

b) wait until the child is clinically well and then vaccinate

c) administer the first vaccination

d) consult a pediatrician

e) none of the above

59) The most common cause of hemoptysis in a 6 year-old boy is:

a) cystic fibrosis

b) pneumonia

c) foreign body

d) tracheobronchitis

e) neoplasm

60) A15 month-old child has spastic cerebral palsy. Which of the

following is most suggestive of perinatal asphyxia as the cause

for this condition:

a) evidence of periventricular calcificatons at birth

b) microcephaly at birth

c) recurrent seizures in the newborn period

d) retinopathy of prematurity

e) rigidity and spasticity

61) Which of the following findings is least consistent with a

diagnosis of congenital deafness in a 16 month-old infant:

a) smiling responsively at 1 month

b) cooing at 2 months

c) babbling at 3 to 6 months

d) imitating speech sounds at 9 months

e) no clearly enunciated words at 15 months

62) Which of the following genetic diseases is incorrectly

linked with its transmission pattern:

a) Duchenne muscular dystrophy = sex-linked recessive

b) pyloric stenosis = polygenetic inheritance

c) cerebral palsy = polygenetic inheritance

d) sickle cell disease = autosomal codominant

e) color blindness = autosomal recessive

70) During the newborn period, physical characteristics suggestive

of prematurity include each of the following except:

a) a prominent diastasis of the rectus muscles

b) scant ear cartilage

c) absent breast buds

d) thin skin

e) hypotonic posture at rest

71) A5 year-old child is diagnosed as having juvenile rheumatoid

arthritis. Which of the following is the least likely side effect of

therapy with a non-steroidal anti-inflammatory drug:

a) abdominal pain

b) lymphadenopathy

c) tinnitus

d) anemia

e) rash

72) Which of the following statements about tumours in childhood

is false:

a) ALL is the most common malignancy

b) brain tumours are the most common solid tumours

c) malignancy is the third most common cause of death after

accidents and homicides

d) neuroblastoma spontaneously regresses

e) blacks and whites do not have equal incidence of tumours

73) Which of the following is a characteristic chest x-ray

appearance of respiratory distress syndrome (RDS):

a) air bronchograms

b) fluid in fissure

c) patchy infiltrate

d) atelectasis

e) enlarged heart

74) An 8 year-old boy presents with sudden onset of pain in the

distal femur. His parents recall that he was hit with a hockey

stick during a street hockey game last week. The most likely

diagnosis is:

a) osteomyelitis

b) fracture of the distal femur

c) juvenile rheumatoid arthritis

d) septic arthritis

e) Blount’s disease

75) The most likely diagnosis in a 3 month-old with a left flank

mass is:

a) renal cell carcinoma

b) Wilm’s tumor

c) neuroblastoma

d) renal stones

e) polycystic kidney disease

76) Which of the following congenital anomalies usually resolves

spontaneously:

a) club foot

b) genu varus

c) tibial torsion

d) flexible flat feet

e) genu valgus

77) Which of the following is the most important factor in

determining whether a child will die of measles:

a) gender

b) age

c) nutritional status

d) muscle mass

e) fitness level

18 Sample Questions Pediatrics Toronto Notes 2005

63) Each of the following is an indication for tonsillectomy except:

a) recurrent episodes of otitis media

b) peritonsillar abscess

c) carcinoma of the tonsil

d) hypertrophy with airway obstruction

e) recurrent (at least six) episodes of group AStrep

tonsillitis within one year

64) Ahealthy, full-term, breastfed infant is jaundiced at 24 hours of

age. The indirect serum bilirubin level is 210 umol/L. The

mother is blood type O, Rh positive. Which of the following is

the most appropriate step:

a) culture the blood

b) determine glucose-6-phosphate dehydrogenase activity

c) interrupt breast feeding

d) obtain a reticulocyte count

e) obtain blood type and Coombs test

65) A6-week-old male infant develops recurrent episodes of

vomiting with dehydration. Each of the following would be

consistent with a diagnosis of congenital hypertrophic pyloric

stenosis except:

a) non-bile stained vomitus after feeding

b) a metabolic acidosis and hyperkalemia

c) a history of increasing constipation

d) good appetite

e) visible epigastric peristaltic waves

66) Each of the following statements regarding AIDS in infants and

children is correct except:

a) the risk of nosocomial transmission is low

b) function of both T and B lymphocytes is impaired

c) hepatosplenomegaly is a common associated finding

d) poor growth occurs frequently

e) chronic pneumonitis is a rare complication

67) A14 year-old boy is concerned about his height. Physical

examination reveals Tanner stage II pubic hair and genital

development. Height and weight are at the 10th percentile for

age. Which of the following is the most accurate statement

about his growth:

a) he is taller than the average 14 year-old girl

b) he will be taller than his peers by age 16

c) he has not reached his peak height velocity

d) he will stop growing by age 17

e) his adult height will be at the 10th percentile

68) A3 year-old girl has a urinary tract infection and receives

appropriate antibiotic therapy. Following treatment,

ultrasonography and voiding cystourethrography reveal normal

findings. Which of the following would be the best

management of this patient's urinary tract problem?

a) administrate polyvalent vaccine against multiple enteric

organisms

b) begin daily prophylaxis now and treat each future episode

of infection

c) obtain intravenous pyelography

d) treat each future episode of infection, but withhold

prophylaxis

e) none of the above

69) A10 year-old boy with cough, fever and night sweats has a

positive skin test for tuberculosis. Achest x-ray reveals minimal

hilar adenopathy and a small pulmonary infiltrate. The current

recommended treatment is:

a) isoniazid alone

b) isoniazid plus rifampin and pyrazinamide

c) isoniazid plus rifampin and streptomycin

d) rifampin and corticosteroids

e) rifampin and ethambutol

Toronto Notes 2005 Pediatrics Sample Questions 19

78) At 36 hours of age a full-term infant has not yet passed

meconium. Physical examination reveals jaundice and

abdominal distention. Radiographs of the abdomen show

dilated loops of bowel and calcifications in the scrotum. A

meconium plug is passed after rectal examination. Which of

the following would be the most definitive diagnostic test to

order:

a) barium enema

b) sweat chloride test

c) ultrasound of the abdomen

d) urine calcium excretion

e) voiding cystourethrogram

79) Organic causes in the differential diagnosis of chronic

abdominal pain includes all of the following except:

a) UTI

b) lactose intolerance

c) chronic giardiasis

d) chronic appendicitis

e) inflammatory bowel disease

80) Which of the following problems is most likely to occur in a

12 month-old infant who began taking whole cow milk at

6 months of age:

a) increased frequency of otitis media

b) wheezing episodes

c) nasal congestion

d) decreased serum ferritin level

e) diaper dermatitis

81) A1 year-old child with a rapid pulse, slightly low blood

pressure, cool extremities, oliguria and dry oral mucosa should

be considered:

a) 5% dehydrated

b) 10% dehydrated

c) 15% dehydrated

d) 20% dehydrated

82) All of the following are indications for tympanostomy tubes

± adenoidectomy in a child except:

a) persistent 3 month effusion

b) failure of prophylactic medications

c) associated hearing loss

d) recurrent acute otitis media

83) A2 year-old with fever is seen in hospital following a

generalized seizure. He also has foul smelling, blood-tinged

diarrhea. The culture of the stool is most likely to grow:

a) Campylobacter species

b) E. coli

c) Entamoeba histolytica

d) Salmonella species

e) Shigella species

9) A32 year-old engineer has been uncharacteristically active for

several weeks. He spends most of his time at work and gets

little sleep. He has told another engineer that he is involved “in

a research project that will earn me the Nobel Prize”.

Expensive research equipment keeps arriving at his office. The

engineer is irritable, and it is hard to hold his attention. A

classmate from graduate school recalls that the patient

behaved in a similar manner twice during stressful periods at

school.

Longterm drug therapy for this patient would likely include:

a) haloperidol

b) valproic acid

c) clozapine

d) ascorbic acid

e) chlordiazepoxide

10) Elderly male depressives typically present with all of the

following EXCEPT:

a) importuning

b) anxiety

c) weight loss

d) little suicide risk

e) insomnia

11) From among the drugs listed below, which would be the cause

for most concern in an overdose:

a) paroxetine (SSRI)

b) amitriptyline (tricyclic)

c) diazepam (benzodiazepine)

d) chlorpromazine (phenothiazine)

e) fluoxetine (SSRI)

12) Which of the following statements about schizophrenia is false?

a) male schizophrenics experience their first psychotic

episode at a younger age than women

b) male schizophrenics are more frequently hospitalized than

female schizophrenics

c) compared to young female schizophrenics, young male

schizophrenics are at increased risk of movement disorders

secondary to neuroleptics

d) in women, the symptoms of schizophrenia tend to worsen

after menopause

e) all of the above statements are true

13) A29 year-old school teacher who lives alone is brought to the

emergency room because she has become increasingly suspicious,

hyperactive, and anorexic over the past two days. She believes

that “people in the neighbourhood are out to get me”. She has

not slept in 2 nights. She reports seeing snakes crawling on the

wall. Based on this information, the most likely diagnosis of the

woman’s problem is:

a) anorexia nervosa

b) cocaine withdrawal

c) paranoid personality

d) psychostimulant abuse

e) shared paranoid disorder

14) Anti-alpha-1-adrenergic blockade causes:

a) nausea

b) constipation

c) orthostatic hypotension

d) dry mouth

e) drowsiness

15) The following are common side effects of SSRIs EXCEPT:

a) Headache

b) Sexual dysfunction

c) Vomiting

d) Anorexia

e) Orthostatic hypotension

20 Sample Questions Psychiatry Toronto Notes 2005

Psychiatry

1) Monoamine oxidase inhibitor drugs are used in the treatment of

depression because they increase synaptic levels of:

a) gamma-aminobutyric acid (GABA)

b) histamine

c) acetylcholine

d) norepinephrine

e) somatostatin

2) Neuropsychological effects of hallucinogens may include all of the

following EXCEPT:

a) miosis

b) tremor

c) hyper-reflexia

d) uncoordination

e) blurred vision

3) Cocaine withdrawal can include all of the following EXCEPT:

a) “Crash” sleep

b) anergia

c) anhedonia

d) euphoria

e) continued craving

4) Alcohol withdrawal includes all of the following EXCEPT:

a) autonomic hyperactivity

b) tremor

c) starts within 2-4 hours after prolonged drinking

d) nausea

e) irritability

5) Which would not be considered a risk factor for suicide in patients

presenting with suicidal ideation:

a) substance abuse

b) male gender

c) lack of social supports

d) unsuccessful attempt at suicide in the past

e) childless marriage

6) A54 year-old man has become forgetful, preoccupied,

withdrawn, irritable and dishevelled. His physical examination

was normal. The patient had been with his company for

twenty-two years and was considered an excellent employee.

Which of the following is the most likely diagnosis:

a) multi-infarct dementia

b) hypothyroidism

c) schizophrenia

d) alcoholism

e) major depression

7) Which of the following is correct about depression in

children:

a) family therapy should be avoided because it scapegoats

a child who is already vulnerable

b) symptoms may manifest as antisocial behaviour

c) antidepressants generally are not effective in children

d) the suicide rate in children aged 8-13 is higher than

it is in older adolescents

e) depression in children has been shown to be a prodrome

to the later development of schizophrenia

8) All of the following are classified as paraphilias EXCEPT:

a) fetishism

b) homosexuality

c) exhibitionism

d) sexual sadism

e) transvestism

Toronto Notes 2005 Psychiatry Sample Questions 21

16) Clozapine is the neuroleptic of choice for schizophrenia when:

a) the patient shows no evidence of tardive dyskinesia

b) the patient has not improved with adequate

dosages for 6 weeks of 3 other antipsychotics

c) the patient is under fifteen years of age

d) the patient has a WBC count of >10 000

e) the patient is very sensitive to anticholinergic side effects

17) Frequent conditions appearing comorbidly with ADHD include:

1) oppositional defiant and conduct disorders

2) anxiety disorder

3) learning disabilities

4) tic disorders

5) language disorders

Which are correct:

a) only 1 is correct

b) 1, 2, and 3 are correct

c) 2 and 4 are correct

d) all are true

18) A35 year-old man presents to the emergency room with

suicidal ideation. He describes significant stress due to

recent job loss and financial hardship. Further inquiry reveals

a history of repeated job loss, fraud charges, and frequent

arm slashing to decrease stress. The man was not disruptive

as a child. The most likely diagnosis is:

a) borderline personality disorder

b) antisocial personality disorder

c) adjustment disorder

d) dysthymic disorder

e) schizophrenia

19) Which of the following statements concerning anorexia

nervosa and bulemia nervosa is false:

a) patients with either of these eating disorders are

preoccupied with weight, food, and body shape

b) bulemia nervosa is more prevalent than is anorexia nervosa

c) both of these eating disorders are more common in

females than in males

d) bulemia nervosa often presents earlier in adolescence than

does anorexia nervosa

e) bulemic symptoms may occur in both bulimia nervosa and

anorexia nervosa

20) Apattern of unstable but intense interpersonal relationships,

impulsivity, inappropriately intense anger, identity disturbance,

affective instability, and problems with being alone suggest a

diagnosis of:

a) antisocial personality disorder

b) narcissistic personality disorder

c) histrionic personality disorder

d) schizoid personality disorder

e) borderline personality disorder

21) The criteria for diagnosis of a factitious disorder include:

a) intentional production or feigning of physical signs or

symptoms

b) absence of secondary gain

c) possibility of economic gain

d) (a) and (b)

e) (a) and (c)

22) Apatient with a fear of heights is brought to the top of a tall

building and required to remain there as long as necessary

for the anxiety to dissipate. This is an example of:

a) graded exposure

b) participant modelling

c) positive reinforcement

d) flooding

e) relationship therapy

23) Hypnosis has been used successfully in all of the following

conditions EXCEPT:

a) pain

b) phobia

c) paranoia

d) anxiety

e) smoking

24) Cognitive therapy helps to correct which of the following

cognitive distortions:

a) depersonalization

b) psychotic thinking

c) over-generalizations

d) selective inference

e) hallucinations

25) The most frequently reported side effect of tricyclic

antidepressants is:

a) peripheral neuropathy

b) photosensitivity

c) agranulocytosis

d) jaundice

e) dry mouth

26) Ahigh risk of suicide is associated with which one of the

following factors:

a) female aged less than 30 years

b) married male less than 30 years

c) public setting

d) secondary gain from attempt

e) single male aged more than 60 years

27) Afixed unalterable belief that is false in its content and in its

social and cultural setting is called:

a) an illusion

b) a hallucination

c) a delusion

d) agnosia

e) paranoia

28) Criteria for involuntary hospitalization of a suicidal patient

may include:

a) acute risk of harm to themself

b) inability to care for self

c) acute risk of harm to others

d) (b) and (c)

e) (a), (b), and (c)

29) Each of the following statements about affective disorders is true

EXCEPT:

a) patients with bipolar disorder show roughly the same

frequency of positive family history as do patients with

unipolar disorder

b) major depressive illness is more common in women than in

men

c) bipolar affective disorder is far less common than unipolar

affective disorder

d) there is a different response to lithium in unipolar and bipolar

disorder

e) imipramine is more likely to produce hypomania in bipolar

patients than in unipolar patients

30) Amajor depressive disorder is diagnosed only when the affective

disturbance has existed for at least :

a) two weeks

b) four weeks

c) six weeks

d) eight weeks

e) twelve weeks

40) Which of the following is best treated with high dose

benzodiazepines:

a) schizophrenia, catatonic type

b) major depression

c) generalized anxiety disorder

d) delirium tremens

e) psychogenic amnesia

41) Personality types predisposed to depression include which

one of the following:

a) dependent

b) antisocial

c) schizoid

d) paranoid

e) schizotypal

42) Which of the following neurological symptoms can be produced

by antipsychotic drugs?

a) akathesia

b) shuffling gait

c) oculogyric crisis

d) tremor at rest

e) all of the above

43) Absolute contraindications to ECT include:

a) pregnancy

b) recent myocardial infarction

c) fractured pelvis

d) brain tumor

e) all of the above

44) In the elderly delirium may be produced by the use of:

a) neuroleptics

b) tricyclic antidepressants

c) antiparkinsonian agents

d) minor tranquilizers

e) all of the above

45) Which of the following statements about simple phobia is correct?

a) it responds well to simple reassurance

b) it responds well to individual psychotherapy

c) it responds well to benzodiazepines

d) it responds well to relaxation and desensitization

e) all of the above

46) Toxicity due to lithium carbonate is associated with all

of the following EXCEPT:

a) nausea

b) serum lithium level of 0.1 mEq/L

c) tremulousness

d) convulsions

47) Which of the following investigations need not be performed

before starting treatment with lithium carbonate?

a) serum creatinine

b) serum electrolytes

c) thyroid function studies

d) serum bilirubin

e) all of the above

48) All of the following are common side effects of tricyclic

antidepressants EXCEPT:

a) dry mouth

b) constipation

c) Parkinsonian tremor

d) tachycardia

49) Which of the following features are seen in dementia due to

cerebral arteriosclerosis:

a) “patchy” amnesia for remote events

b) more common in women than men

c) death occurs frequently due to cerebrovascular

accidents

d) increased “senile plaque” count

e) all of the above

22 Sample Questions Psychiatry Toronto Notes 2005

31) A30 year-old man presents in emergency with right lower quadrant

abdominal pain. His wife reports that he had been drinking heavily

in response to marital problems and had never had such pain

before. Appendicitis was diagnosed and an appendectomy was

successfully performed. Four days later the patient was anxious,

restless, unable to sleep and claimed his wife was a stranger trying

to harass him. The likeliest diagnosis is:

a) paranoid reaction

b) delirium tremens

c) mania

d) schizophreniform reaction

e) post-operative delerium

32) Which of the following symptoms of schizophrenia responds best to

anti-psychotic medication:

a) anti-social behavior

b) flat affect

c) paranoid delusions

d) lack of motivation

e) all of the above

33) A53 year-old housewife presents with depression marked by early

morning wakening, diminished energy and poor concentration. She

is treated with supportive psychotherapy and celexa

20 mg. After four weeks she is no better. You would next:

a) reassure her that antidepressants take 5-6 weeks to be

effective

b) begin intensive psychotherapy

c) switch to a different SSRI

d) increase the dose to 40 mg

e) (a) and (d)

34) Hypertensive encephalopathy is a serious complication of treatment

with:

a) phenothiazines

b) tricyclic antidepressants

c) lithium carbonate

d) MAOI antidepressants

e) benzodiazepines

35) Electroconvulsive therapy (ECT) is a treatment in

which one of the following conditions?

a) obsessive compulsive disorder

b) paranoid schizophrenia

c) generalized anxiety disorder

d) acute mania

e) major depression with psychotic features

36) Tardive dyskinesia is:

a) an acute extrapyramidal side effect of antipsychotic

b) an acute anticholinergic side effect of tricyclic antidepressants

c) a type of Parkinson’s disease

d) a side effect of excessive ECT treatments

e) a long term complication of chronic antipsychotic

administration

37) Which of the following symptoms is most commonly found in

schizophrenia?

a) depressed mood

b) flight of ideas

c) elevated mood

d) delusional fears

e) thought insertion

38) Patients with conversion disorders will show each of the following

EXCEPT:

a) “la belle indifference”

b) severe depression

c) loss of special sense function

d) secondary gains

e) paralysis of voluntary muscles

39) The syndrome of delirium tremens is associated with each of the

following EXCEPT:

a) rapidly fluctuating level of consciousness

b) dehydration

c) visual hallucinations

d) renal failure

e) polyneuropathy

Toronto Notes 2005 Psychiatry Sample Questions 23

50) The Theory of Behavioral Therapy employs which of the following

concepts:

a) reaction formation

b) reinforcement

c) imprinting

d) autosuggestion

e) sublimation

51) Which of the following represents a contraindication for

psychoanalysis?

a) the existence of transference feeling toward the analyst

b) the existence of countertransference feeling in the analyst

c) the resistance to change by the patient

d) preoccupation with resolving an acute crisis situation

e) the existence of an underlying neurotic style of

behaviour

52) Which of the following is least helpful in treating depression?

a) phenelzine

b) imipramine

c) lithium carbonate

d) lorazepam

e) all of the above

53) Ayoung mother is very focused on the health of her

16 month-old. She keeps her house immaculate for fear that

dirt will harm her baby, she checks the lock on the door at

least ten times before retiring to bed, and she has to get up

and check that her child is still breathing at least 3 times

every night. She knows that her fears are irrational but

persists with these behaviors. The most likely diagnosis is:

a) paranoid delusions not otherwise specified

b) post-partum depression

c) obsessive compulsive disorder

d) generalized anxiety disorder

e) paranoid personality disorder

54) Athorough assessment for the presence/absence of alcohol

withdrawal should include questions about all of the following

EXCEPT:

a) nausea and vomiting

b) mood

c) difficulty walking (ataxic gait)

d) visual disturbances

e) tremulousness

55) Which of the following will be least helpful with respect to

distinguishing delerium from dementia:

a) disorientation at night

b) duration of disorientation

c) fluctuating level of consciousness

d) mini mental status examination

e) presence of visual hallucinations

10) Which of the following is not a sign or symptom of carpal tunnel

syndrome :

a) loss of sensation to the proximal palm

b) positive Phalen test

c) positive Tinel sign

d) pain involving the thumb, index, long, and part of the ring

digits

e) delayed transit time on nerve conduction study

11) A35 year-old intoxicated man presents to the Emergency

Department with a deep laceration to his right distal forearm after

putting his hand through a window. On examination, he has no

sensation to his little finger and the ulnar aspect of his ring finger.

Power on abduction and adduction of all fingers is markedly

decreased. He is unable to flex the distal joint of his little finger.

When he flexes his wrist, his hand deviates radially. The

examination was otherwise normal. What structure(s) have been

damaged?

a) ulnar nerve

b) flexor carpi ulnaris and flexor digitorum profundus muscles

c) median nerve

d) radial nerve

e) brachioradialis

12) In which of the following are systemic antibiotics not indicated :

a) animal bite to hand

b) laceration over metacarpal-phalangeal joint sustained after

punching someone in a fight

c) positive Finkelstein test

d) burn wound to 75% of the dorsal aspect of the hand

e) acute suppurative tenosynovitis

13) Prolonged vomiting is associated with what electrolyte

abnormality?

a) hypochloremic hypokalemic metabolic acidosis

b) hypochloremic hypokalemic metabolic alkalosis

c) hyperchloremic metabolic acidosis

d) hyperkalemia

e) none of the above

14) Indications for immediate operative intervention rather than

conservative management for arterial insufficiency include all the

following EXCEPT:

a) nocturnal limb pain

b) ischemic ulceration

c) absent pulse

d) ischemic neuropathy

e) toe gangrene

15) Surgical indications for diverticulitis include all of the following

EXCEPT:

a) peritonitis

b) persisting hemorrhage

c) fistula

d) greater than two severe attacks

e) palpable abdominal mass in left lower quadrant

16) Six days following a hemicolectomy for colorectal carcinoma, a 54

year-old woman experiences calf pain unilaterally. On physical

examination, the affected side has a greater diameter than the

other, and there is tenderness to palpation. Which of the

following investigations would be the next step:

a) venogram

b) compression venous Doppler flow studies

c) x-ray of the affected calf

d) ventilation/perfusion scan

e) compartment pressure monitoring

17) All of the following radiologic abdominal plain film findings are

consistent with mechanical bowel obstruction except:

a) a "step ladder" pattern

b) dilated small bowel loops

c) air-fluid levels at uniform height in same bowel loop

d) absence of gas in large bowel

e) rows of small gas accumulations in valvulae conniventes (i.e.

"string of pearls")

24 Sample Questions Surgery Toronto Notes 2005

Surgery

1) A25 year-old known substance abuser is brought to the ED with a

suspected overdose. Which of the following is not considered a

universal antidote?

a) glucose

b) oxygen

c) calcium gluconate

d) naloxone

e) thiamine

2) Which of the following is not a classic sign of a basal skull fracture?

a) Battle sign

b) racoon eyes

c) hemotympanum

d) Freedman sign

e) CSF rhinnorhea/ottorrhea

3) A19 year-old female with a traumatic head injury is brought to the ED

by EMS. She is hemodynamically stable but requires assisted

ventilation through an endotracheal tube. She does not open her eyes

to painful stimuli and maintains an abnormal extension posture.

Which of the following is incorrect?

a) her GCS is 3T

b) a GCS of 8 is an indication for intubation

c) isolated head injuries can cause shock

d) she should be ventilated to a pCO2 of 30-35 mm Hg

e) all are incorrect

4) Conductive hearing loss is a symptom of:

a) presbycusis

b) Meniere disease

c) cholesteatoma

d) Bell palsy

e) acoustic neuroma

5) The following statements regarding epistaxis are false EXCEPT:

a) Epistaxis rarely occurs in children

b) It commonly results from rupture of posterior placed nasal vessels

c) It may be treated by ligation of the ipsilateral internal carotid

artery

d) Epistaxis may be treated by cautery of Little's area with silver

nitrate

e) It is not a common cause of emergency admission to ENT wards

6) A52 year-old dentist comes to your office complaining of severe

bilateral buttock cramps and thigh fatigue during a tennis match and

recent onset of impotence The most likely diagnosis is:

a) lumbosacral disc problem

b) multiple sclerosis

c) Leriche syndrome

d) metastatic carcinoma of the spine

7) Ayoung woman who was involved in a cliff diving accident is

brought into the emergency department unresponsive to deep pain

with a right pupil that is dilated and nonreactive. The left pupil is

normal. The most appropriate inital treatment is:

a) endotracheal intubation

b) CT scan of the head

c) 500 cc normal saline IV

d) IV steroids

e) IV mannitol

8) Patients with an organic cause for impotence are often characterized

by all of the following EXCEPT:

a) diabetes

b) older age

c) intermittent difficulty

d) nocturnal penile tumescence absent

e) none of the above

9) In the investigation of infertility, a normal semen analysis MUST

contain:

a) a volume of 0.5-1 mL

b) morphology > 85% normal forms

c) WBC < 1 per high power field

d) motility > 75%

e) sperm count > 20 million sperm/mL

Toronto Notes 2005 Surgery Sample Questions 25

18) After a weekend of heavy drinking, a 28 year-old male presents with

abdominal pain radiating to the back. What is the investigative and

prognostic modality of choice for the suspected diagnosis?

a) abdominal ultrasound

b) abdominal CT

c) abdominal plain film

d) endoscopic retrograde cholangiopancreatography

e) laparoscopy

19) An 18 year-old motorcyclist presents in the emergency department

following an accident. He has a compound tibia and fibula fracture

of the right leg and on examination the right leg has no pulses. Your

immediate treatment should be:

a) immediate angiogram

b) immediate surgery

c) casting and/or splinting

d) reduction and splinting

e) x-ray

20) Which of the following is the most serious complication of a dis

placed supracondylar fracture of the humerus?

a) compartment syndrome of the forearm

b) failure to heal

c) healing in a non-anatomical position

d) injury to the median nerve

e) significantly limited range of elbow motion

21) What is the typical audiogram finding in noise induced hearing loss:

a) conductive loss in the low frequencies

b) conductive loss at 2000 Hz

c) sensorineural loss at 2000 Hz

d) sensorineural loss at 4000 Hz

e) mixed broad spectrum hearing loss

22) Which of the following is not a complication of untreated otitis

media?

a) cholesteatoma

b) meningitis

c) tympanic membrane perforation

d) trigeminal neuralgia

e) conductive hearing loss

23) With respect to control of micturition, all of the following are true

EXCEPT:

a) damage to the cerebral cortex results in hyperactivity of the

detrusor

b) the basal ganglia inhibits bladder overactivity

c) the cerebellum coordinates emptying of the bladder

d) damage to the cerebellum results in hypotonicity of the detrusor

e) injury above the brainstem results in hyperactive bladder only

24) Which of the following drugs will not promote urine retention?

a) sympathomimetics

b) anticholinergics

c) cholinergic agonists

d) TCAs

e) smooth muscle depressants

25) A28 year-old male presents with painless swelling in the testicle.

On examination, it appears to be a rubbery, hard mass. He has no

palpable abdominal masses. Atesticular ultrasound confirms the

diagnosis of testicular tumour, and an abdominal CT reveals normal

retroperitoneal nodes. Which statement is correct with respect to this

case?

a) his overall prognosis is very poor

b) the testicular mass is likely a secondary tumour

c) a needle aspiration of the tumour is indicated

d) a transcrotal approach for biopsy is contraindicated

e) he will likely require chemotherapy

26) A78 year-old woman complains of experiencing headaches and

progressive confusion for the last month. She has a left hemianopia

and cannot dress herself. ACT scan demonstrates a large, irregularly

enhancing mass in the right parietal lobe. There is no obvious

systemic disease. The most likely diagnosis is:

a) brain abscess

b) glioblastoma multiforme

c) meningioma

d) metastasis

e) CNS lymphoma

27) Apatient with a subarachnoid hemorrhage (SAH) caused by a right

anterior communicating artery aneurysm undergoes successful

surgery 2 days after the hemorrhage. Three days later, right arm

weakness develops. The most likely diagnosis is:

a) hydrocephalus

b) meningitis

c) repeat hemorrhage

d) vasospasm

e) none of the above

28) Complications of tonsillectomy include all of the following EXCEPT:

a) secondary hemorrhage

b) severe otalgia

c) Quinsy

d) nasopharyngeal stenosis

e) none of the above

29) Features characteristic of acute tonsilitis include all of the following

EXCEPT:

a) odynophagia

b) cough

c) referred otalgia

d) cervical lymphadenopathy

e) dysphagia

30) A6 month-old baby has had mild inspiratory stridor for the last

2 months. The most likely cause of this is:

a) laryngomalacia

b) acute epiglottitis

c) croup

d) tonsillar hypertrophy

e) foreign body aspiration

31) A48 year-old male is brought to the ED with CPR being

administered. The ECG shows electrical activity present but you

cannot palpate a carotid pulse. Which of the following is not on the

differential for pulseless electrical activity?

a) hypotension

b) hypokalemia

c) cardiac tamponade

d) hypothermia

e) tension pneumothorax

32) A37 year-old male arrives at the Emergency Department

unconscious. He is warm and sweaty. His heart rate is 52 bpm, his

BPis 90/60. His pupils are constricted, his eyes are teary, and he is

drooling. You assume he is suffering from a toxidrome. What

antidote will you give him?

a) Flumazenil

b) Naloxone

c) Glucagon

d) Atropine

e) Ethanol

33) A60 year-old man presents with back and leg pain and trouble

urinating. On examination, he has decreased sensation over the

buttocks, normal motor power, and absent ankle jerks bilaterally.

How would this patient best be investigated?

a) plain films of the lumbar spine

b) CT of lumbar spine

c) MRI of spine

d) investigate only if no improvement after 6 weeks of symptomatic

treatment

e) gallium scan of the spine

34) A34 year-old woman presents with a 3 day history of increasing

fever with chills, sweats, nausea, and 2 episodes of vomiting. On

examination, she has costovertebral angle tenderness and urine

reveals gross pyuria. Initial management should include:

a) abdominal ultrasound

b) abdominal CT

c) empirical treatment with antibiotics; image only if she fails to

respond to therapy

d) treat with antibiotics and perform IVP 4-6 weeks after resolution

of illness

e) immediate DTPArenal scintigraphy

43) Which of the following is not a surgically correctable cause of

hypertension?

a) aortic coarctition

b) renal artery atherosclerosis

c) pheochromocytoma

d) primary hyperaldosteronism (Conn syndrome)

e) none of the above

44) A16 year-old female fell while roller-blading on her outstretched

right hand. At a nearby emergency department X-rays confirmed

the diagnosis of a closed Colles fracture. The proper reduction

technique for this wrist fracture is which of the following:

a) slight extension, full pronation, and full ulnar deviation

b) slight flexion, full supination, and full radial deviation

c) slight extension, full supination, and full ulnar deviation

d) slight flexion, full pronation, and full ulnar deviation

e) slight extension, full pronation, and full radial deviation

45) Which of the following is least likely to cause avascular necrosis:

a) sickle cell disease

b) septic arthritis

c) steroid use

d) constrictive dressings

e) post-traumatic fracture

46) A24 year-old football player severely fractures his ankle while

playing in a game and subsequently requires ORIF treatment.

Indications for ORIF treatment of an ankle fracture include all of

the following except:

a) a fracture-dislocation

b) undisplaced fracture with Grade II ATFL tear

c) trimalleolar fracture

d) unstable talar tilt

e) unable to maintain a closed reduction

47) Which of the following is not an indication for repair of an orbital

blow-out fracture?

a) enopthalmos

b) persistent diplopia

c) positive forced duction test

d) orbital floor lesion < 0.7 cm

e) other upper facial fractures

48) Which of the following is true regarding mandibular fractures?

a) they are predominantly unilateral

b) compound fractures are rarely intraoral

c) they may manifest with numbness in the V2 facial nerve

distribution

d) malocclussion of teeth is a common sign

e) they seldom result in trismus

49) With respect to painful scrotal swelling, all of the following are

true EXCEPT:

a) torsion of testicular appendages will usually subside without

surgical intervention

b) nausea/vomiting is very common in epididymitis

c) ultrasound is helpful in determining the cause of hematocele

d) torsion most commonly occurs in young males

e) orchitis may result in testicular atrophy

50) Which is following statement about PSA is true?

a) every man past the age of 50 should have an annual PSA test

as a primary screen for prostate cancer

b) PSA is best used to follow disease progression or recurrence

post-operatively

c) PSA is a useless test

d) increased complex PSA to total PSA ratio favors BPH over

prostate cancer

e) all of the above

51) A40 year-old women presents to the ER with fever, no nausea or

vomiting, and left flank pain radiating to the groin. Body CT

reveals 8 mm stone in the left proximal ureter. You should:

a) send her home because renal stone is benign disease

b) send her home with oral analgesics and ask her to come back if

the pain does not resolve in the next 24 hours

c) admit, close monitoring, hydration and analgesics

d) admit, hydration, analgesics and IV antibiotics. (e.g. Amp and

Gent), and consider stenting if the symptom persists

26 Sample Questions Surgery Toronto Notes 2005

35) All of the following statements regarding knee injuries are correct

EXCEPT:

a) locking of the knee may be due to a torn meniscus

b) minor tears of the medial collateral ligament can be treated

with brief immobilization then range of motion and

strengthening exercises

c) Lateral meniscus tears are more common than medial meniscal

tears

d) anterior cruciate ligament tears may give a positive Lachman

test

e) a knee dislocation may be associated with major ligament

damage

36) An 83 year-old man has fallen while walking down stairs. He is

brought to the emergency department with a 3-part

intertrochanteric hip fracture. Which of the following procedures

would you choose to perform?

a) hemiarthoplasty

b) total hip replacement

c) multiple pin fixation

d) bipolar arthroplasty

e) pin and plate

37) A45 year-old man with a history of polycystic kidney disease

presents with painless gross hematuria. You order:

a) no investigation is required since the hematuria is most likely

due to the rupture of renal cyst(s)

b) no investigation at this time. Investigate if gross hematuria

persists

c) U/S

d) U/S, urine C&S, cystoscopy

38) A75 year-old man with a history of nocturia has not micturated

for the last 10 hours, and is complaining of severe lower abdominal

pain. The most likely cause is:

a) BPH

b) prostate cancer

c) renal failure

d) UTI

e) prostatitis

39) A43 year-old woman is brought to the Emergency Department

after being burned in a house fire. You estimate first degree burns

to 20% of her body, second degree burns to 11% of her body, and

third degree burns to 9% of her body. She weighs 60 kg and is

120 cm tall. What IV therapy would you begin immediately?

a) normal saline at 200 cc/hr for 24 hours

b) normal saline at 400 cc/hr for 24 hours

c) normal saline at 150 cc/hr for 8 hours, then 75 cc/hr for the

next 16 hours

d) normal saline at 300 cc/hr for 8 hours, then 150 cc/hr for the

next 16 hours

e) normal saline at 600 cc/hr for 8 hours, then 300 cc/hr for the

next 16 hours

40) Which of the following statements is true?

a) skin grafts have their own blood supply

b) granulation tissue, bone, and perichondrion can all support a

skin graft

c) wound contracture does not occur with skin grafting

d) axial flaps do not rely on a particular anatomically defined

vascular bundle

e) muscle flaps can aid in immunologic defense in infected

wounds

41) Risk factors for hepatocellular carcinoma include all of the

following EXCEPT:

a) hepatitis A

b) cirrhosis

c) exogenous steroid use

d) hemochromatosis

e) smoking

42) Which of the following is associated with biliary colic?

a) epigastric pain

b) rebound tenderness

c) jaundice

d) Murphy sign

e) all of the above

Toronto Notes 2005 Surgery Sample Questions 27

52) Nasopharyngeal carcinoma:

a) is most common in people from south-east Africa

b) presents early with nasal pain

c) is treated by wide surgical excision

d) is monitored by measurement of Epstein-Barr virus antibodies

e) is curable in 90% of patients

53) Factors which contribute to the development of squamous cell

cancers of the head and neck include all of the following EXCEPT:

a) cigarette smoking

b) alcohol ingestion

c) aging

d) exposure to particular emission from diesel engines

e) solar irradiation

54) Which of the following statements about aspiration of a peanut

into the tracheobronchial tree is FALSE?

a) the peanut is more likely to be in the left lower lobe bronchus.

b) expiratory wheeze is the most likely finding on physical exam

c) the peanut should be removed with the patient under general

anesthesia through an open bronchoscope with forceps

designed to grasp peanuts

d) a chest x-ray may show atelectasis distal to the blocked

bronchus

e) pneumonia may be a complication

55) Which of the following is an absolute indication for a

tonsillectomy?

a) airway obstruction

b) recurrent (>5) episodes of tonsillitis

c) peritonsillar abscess

d) tonsillar hypertrophy

e) halitosis

56) A72 year-old man on physical examination is found to have

expressive dysphasia and mild right arm weakness. The most

probable location of his lesion is:

a) right parietal lobe

b) left frontal lobe

c) right frontal lobe

d) left parietal lobe

e) basal ganglia

57) Inital management of any patient with coma of undetermined

cause includes all except:

a) clear and secure the airway

b) naloxone

c) D50W 50 mL IV

d) dexamethasone 16 mg IV

e) thiamine

58) A53 year-old male presents to the ED with new onset of a severe

headache associated with nausea and vomiting. There is no

history of trauma. He is alert and oriented with no neck stiffness.

Anisocoria is present. The most likely diagnosis is:

a) intracranial bleed

b) cluster headache

c) meningitis

d) migraine headache

e) tension headache

59) A16 year-old girl is brought to hospital by her frantic parents after

a bee sting. Vitals sings are BP 70/40, RR 30 and laboured, HR 140,

T 37.5. Which of the following would not be an option in her

management?

a) epinephrine

b) diphenhydramine

c) methylprednisolone

d) salbutomol

e) atropine

60) A24 year-old woman arrives at the Emergency Department

unconscious. Her BP is 90/60, her heart rate is 60 bpm, she is

breathing at 8 breaths per minute and her O2 sat is 86%. Her

eyes remain closed even after pain stimulation and the only sounds

she makes are incomprehensible. Her elbows and wrists are flexed

with her feet extended. This patient’s GCS score is:

a) 3

b) 4

c) 5

d) 6

e) 7

61) Which physical exam finding below is usually not associated with

increasing intracranial pressure?

a) deteriorating level of consciousness

b) increasing heart rate and increasing blood pressure

c) yawning, hiccuping, vomiting

d) seizures

e) unilateral sixth cranial nerve palsy

62) Which of the following is false regarding post operative wound

infections?

a) S. aureus is the most common cause

b) usually present with fever post of day 3-4

c) increased likelihood in diabetics

d) mainly treated with antibiotics

e) risk increases with length of surgery

63) Which of the following signs and symptoms warrant surgical

intervention for patients with small bowel obstruction?

a) abdominal tenderness

b) air-fluid levels on abdominal x-ray

c) worsening abdominal pain

d) feculent vomitus

e) air in the colon and rectum on abdominal x-ray

64) A59 year-old woman presents to her family physician with a 3 cm

palpable, well circumscribed, non-tender breast mass. She first

noticed it several months ago and believes it has increased in size

since then. Her mother and maternal aunt were diagnosed with

breast cancer in their early 50's. She has no other health complaints.

Asubsequent mammogram shows no abnormalities. Which of the

following is the next most appropriate step in management?

a) prophylactic mastectomy

b) repeat mammogram in 6 months

c) repeat mammogram ion 1 year

d) excisional biopsy

e) ultrasound

65) Which of the following is false regarding pancreatic pseudocysts?

a) it is caused by duct leakage

b) clinically suspected if persisting pain > 2 weeks following

diagnosis of acute pancreatitis

c) majority are treated surgically

d) surgical intervention is typically delayed to allow pseudocyst to

mature

e) lacks true epithelium

66) An overweight, 45 year-old man presents with left lower quadrant

tenderness and a one week history of abdominal pain, loose nonbloody

stools and worsening fever. Laboratory investigation yields

leukocytosis with neutrophilia and left shift. What is the diagnostic

modality of choice?

a) CT scan

b) barium enema

c) ultrasound

d) plain abdominal film

e) angiography

67) A19 year-old woman is brought to the emergency room following

involvement in a motor vehicle accident. On examination, she has a

GCS of 10 and swelling over the occipital protuberance. The most

appropriate imaging study is:

a) MRI of skull and contents

b) skull films

c) head CT with contrast

d) head CT without contrast

e) CT myelogram

28 Sample Questions Surgery Toronto Notes 2005

68) An 80 year-old woman suffers cervical spinal soft-tissue injury in a

motor vehicle accident with no skeletal or neurologic damage

documented at the time. Three months later, she presents with

sudden onset of homonymous right upper quadrantanopia. CT

demonstrates a non-hemorrhagic lesion in the left lower occipital

lobe. Which imaging study would likely yield the most useful

information?

a) carotid Doppler ultrasound

b) echocardiography

c) MR angiography

d) SPECT scan

e) C-spine plain films

69) Which of the following radiographic features is most consistent

with osteoarthritis of the knee?

a) marginal erosions

b) juxta-articular osteopenia (demineralization)

c) loss of articular cartilage with narrowing of the radiologic joint

space

d) osteonecrosis (avascular necrosis) of the medial femoral condyle

e) syndesmophyte formation

70) Which of the following statements is incorrect with respect to

wound healing?

a) epithelialization can occur within 24 hours following primary

closure of a wound

b) maximum wound strength is often achieved after 2 years

c) wounds continue to gain strength after collagen synthesis has

reached an equilibrium

d) wound contracture is mediated by myofibroblasts

e) the incidence of wound infection increases with healing by

secondary intention

71) Which of the following is not a cause of sensorineural hearing loss:

a) ossicular discontinuity

b) ototoxicity

c) Méniére disease

d) noise

e) presbycusis

72) Which of the following is not true of Meniere’s disease?

a) characterized by quadrad of vertigo, hearing loss, tinnitus and

aural fullness

b) vertigo burns out with time

c) can be treated with diuretics

d) it is the result of an abnormal buildup of potassium

e) the vertigo lasts for seconds

73) Aneurosurgeon complains of a 3 week history of awakening at

night with right-hand discomfort that resolves after several minutes.

On examination, he has mild weakness of thumb abduction and

diminished pain sensibility on the palmar aspect of the thumb and

index finger. The most likely diagnosis is:

a) carpal tunnel syndrome

b) cervical radiculopathy

c) reflex sympathetic dystrophy

d) tendonitis

e) left middle cerebral artery ischemic attacks

74) A73 year-old woman presents with a 6 month history of

deteriorating gait and low back discomfort, exacerbated by walking.

Examination is unremarkable except for hypoactive muscle stretch

reflexes in the legs. X-rays of the lumbosacral area shows the

expected degenerative changes associated with a woman of her age.

The most likely diagnosis is:

a) acute lumbar disc hernation

b) lumbar stenosis

c) myopathy

d) normal pressure hydrocephalus

e) cervical stenosis

75) Which of the following findings is inconsistent with cardiac

tamponade?

a) hypotension

b) pulsus paradoxus

c) Kussmaul sign

d) jugular venous distension

e) muffled heart sounds

76) Tension pneumothorax is best diagnosed with:

a) stat CT scan

b) chest x-ray

c) watch and wait

d) clinical exam

e) none of the above

77) Which of the following is NOT a cause of major lower

gastrointestinal hemorrhage?

a) diverticulitis

b) angiodysplasia

c) aortoenteric fistula

d) none of the above

78) All of the following have been associated with posterior shoulder

dislocation EXCEPT:

a) ethanol

b) electricity

c) exercise

d) epilepsy

e) encephalitis

79) Management of an open fracture should always include each of the

following EXCEPT:

a) assessment of neurovascular status

b) reduction and fixation of fracture

c) irrigation and debridement of wound

d) application of sterile dressing

e) application of topical antibiotic

80) What is the most common type of thyroid cancer?

a) medullary

b) papillary

c) follicular

d) lymphoma

e) anaplastic

Toronto Notes 2005 Answers to Sample Questions Sample Questions 29

Answers to Sample Questions

COMMUNITY AND PUBLIC HEALTH

1. D 14. E 27. D 40. A 53. C

2. D 15. D 28. E 41. C

3. C 16. B 29. D 42. D

4. B 17. D 30. B 43. C

5. C 18. A 31. D 44. B

6. C 19. D 32. B 45. A

7. E 20. A 33. D 46. E

8. E 21. E 34. B 47. B

9. D 22. B 35. D 48. C

10. B 23. C 36. B 49. E

11. A 24. E 37. C 50. C

12. B 25. D 38. A 51. D

13. C 26. E 39. B 52. B

MEDICINE

1. C 16. C 31. B 46. A 61. C 76. E

2. B 17. B 32. E 47. D 62. D 77. D

3. D 18. E 33. D 48. D 63. D 78. C

4. A 19. D 34. C 49. D 64. B

5. E 20. E 35. C 50. D 65. D

6. C 21. E 36. B 51 D 66. E

7. C 22. D 37. F 52. D 67. C

8. E 23. E 38. C 53. B 68. B

9. D 24. D 39. A 54. A 69. C

10. B 25. B 40. B 55. B 70. D

11. D 26. C 41. B 56. C 71. B

12. B 27. A 42. A 57. E 72. B

13. B 28. D 43. E 58. A 73. E

14. A 29. C 44. D 59. B 74. B

15. C 30. C 45. D 60. E 75. C

OBSTETRICS AND GYNECOLOGY

1. C 13. A 25. E 37. C 49. E

2. B 14. E 26. A 38. B 50. D

3. D 15. C 27. C 39. C

4. C 16. C 28. D 40. E

5. D 17. D 29. E 41. B

6. C 18. B 30. C 42. D

7. E 19. B 31. B 43. B

8. C 20. C 32. D 44. B

9. E 21. E 33. D 45. B

10. B 22. C 34. D 46. D

11. E 23. B 35. C 47. B

12. B 24. E 36. A 48. A

30 Sample Questions Answers to Sample Questions Toronto Notes 2005

PEDIATRICS

1. E 17. B 33. C 49. E 65. B 81. B

2. A 18. B 34. C 50. D 66. E 82. D

3. A 19. B 35. D 51. A 67. C 83. E

4. C 20. B 36. A 52. B 68. E

5. A 21. A 37. C 53. D 69. B

6. C 22. A 38. C 54. E 70. A

7. B 23. E 39. A 55. A 71. B

8. E 24. B 40. C 56. E 72. D

9. D 25. D 41. E 57. D 73. A

10. A 26. C 42. A 58. C 74. A

11. B 27. B 43. B 59. C 75. B

12. E 28. C 44. D 60. C 76. B

13. D 29. A 45. C 61. D 77. C

14. B 30. B 46. A 62. E 78. B

15. A 31. B 47. B 63. A 79. D

16. B 32. E 48. B 64. E 80. D

PSYCHIATRY

1. D 15. E 29. A 43. D

2. A 16. B 30. A 44. E

3. D 17. D 31. B 45. D

4. C 18. A 32. C 46. B

5. E 19. D 33. E 47. D

6. E 20. E 34. D 48. C

7. B 21. D 35. E 49. C

8. B 22. D 36. E 50. B

9. B 23. C 37. D 51. D

10. D 24. C 38. B 52. D

11. B 25. E 39. D 53. C

12. E 26. E 40. D 54. C

13. D 27. C 41. A 55. A

14. C 28. E 42. E

SURGERY

1. C 15. E 29. B 43. D 57. D 71. A

2. D 16. B 30. A 44. D 58. A 72. E

3. C 17. C 31. B 45. B 59. E 73. A

4. C 18. B 32. D 46. D 60. D 74. B

5. D 19. D 33. C 47. D 61. B 75. C

6. C 20. A 34. C 48. D 62. D 76. D

7. A 21. D 35. C 49. B 63. C 77. A

8. C 22. D 36. E 50. B 64. D 78. C

9. E 23. D 37. D 51. D 65. C 79. E

10 A 24. C 38. A 52. D 66. A 80. B

11. A 25. D 39. D 53. D 67. D

12. D 26. B 40. E 54. A. 68. C

13. B 27. D 41. A 55. A 69. C

14. C 28. C 42. A 56. B 70. E

Toronto Notes 2005 Key Feature Questions Sample Questions 31

CASE1

A30 year-old homeless male presents with a 4 week history of feeling

generally unwell with anorexia, weight loss and intermittent fever. He

lives primarily on the streets or occasionally in a men’s shelter. He

admits to regular use of intravenous heroin for the last seven years and

often shares needles.

QUESTION 1 (CASE 1)

Given this history, which of the following diagnoses would you consider?

Select up to five.

1. Acute pancreatitis

2. AIDS related complex

3. Aspergillosis

4. Bacterial endocarditis

5. Chronic pancreatitis

6. Delirium tremens

7. Diabetes mellitus

8. Hepatitis B

9. Hepatocellular carcinoma

10. Histoplasmosis

11. Leukemia

12. Lymphoma

13. Osteomyelitis

14. Peptic ulcer disease

15. Pericarditis

16. Pyelonephritis

17. Renal failure

18. Secondary syphilis

19. Talc pneumonitis

20. Tuberculosis

QUESTION 2 (CASE 1)

Amitral regurgitant murmur is auscultated in this patient.

His temperature is 39.0ºC.

Which of the following other findings may be found on physical

examination given the diagnoses being considered?

Select up to five.

1. Ascites

2. Campbell De Morgan’s spots

3. Cannon “a” waves

4. Cyanosis

5. Erythema nodosum

6. Follicular keratosis

7. Gynecomastia

8. Hepatomegaly

9. Janeway lesions

10. Jaundice

11. Koilonychia

12. Livedo reticularis

13. Osler nodes

14. Pallor

15. Palmar erythema

16. Papilledema

17. Purpura

18. Roth spots

19. Spider nevi

20. Splenomegaly

Key Feature Questions

Instructions for Examinees

“Key Feature” questions can pertain to solutions of clinical cases that involve data-gathering (e.g., history taking, physical examination, laboratory

investigations), diagnosis or treatment.

The Key Feature questions will ask for your answers in different ways: you may be asked to (i) select the correct response(s) from a menu of options, or

(ii) print the correct answer(s) on a blank line.

Please note the following guidelines before proceeding:

1. After reading the description of the case, read the question carefully. Take note as to whether you are told the maximum number of answers to

provide. If you exceed this number, you will receive no credit for the question. If you select or list fewer answers, you will still receive credit

for any of your answers which are correct.

2. If you are asked to select your answers from a menu:

a. Read the complete menu of options before recording your answers.

b. Make certain that the number of answers you record is not greater than the number you have been asked to select.

3. If you are asked to list your answers below the question:

a. Ensure that you print each answer legibly.

b. Record each answer on a separate blank line.

c. Do not record more than the maximum number you are asked to list.

CASE 2

Ababysitter brings a 14 month-old to emergency because of inconsolable

crying. The child appears well cared for and healthy but a

"bruise" is noted on the left humerus and a well-circumscribed small

fresh burn is noted on the other shoulder.

QUESTION 3 (CASE 2)

List four well recognized risk factors for child abuse:

1. _____________________________________________________________

2. _____________________________________________________________

3. _____________________________________________________________

4. _____________________________________________________________

QUESTION 4 (CASE 2)

Which of these physical findings are most suggestive of child abuse?

Select up to four.

1. Alopecia

2. Blue sclera

3. Bruises on shins and elbows

4. Buttocks wasting

5. Caput medusa

6. Clubbing

7. Condylomata acuminata

8. Eczema

9. Full fontanelle

10. Healed laceration on chin

11. Hutchison teeth

12. Impetigo

13. Large purple macular lesion on buttock

14. Limp

15. Oral thrush

16. Petechiae

17. Pitted finger nails

18. Proptosis

19. Retinal hemorrhage

20. Seborrhea

QUESTION 5 (CASE 2)

You suspect this is a case of child abuse. Which of the following are

appropriate steps in the initial management. Select up to four.

1. Abdominal ultrasound

2. Ascorbic acid level

3. Bone density studies

4. CBC

5. Chest x-ray

6. Coagulation studies

7. Creatinine

8. Dietary history

9. ECG

10. Factor 8 assay

11. Liver spleen scan

12. MRI

13. Notify CAS (Children’s Aid Society)

14. Notify police

15. Notify public health nurse

16. Ophthalmology consult

17. Psychiatry consult

18. Serum calcium, phosphorus and alkaline phosphorus

19. Skeletal survey

20. Urinalysis

CASE 3

An 84 year-old female is brought to the Emergency Department by her

daughter who states that her mother has not been herself over the past

three days. Further questioning reveals that the mother has been less

attentive than usual and more withdrawn, her conversation is often

rambling and she has not been sleeping well. Prior to this, she functioned

very well.

QUESTION 6 (CASE 3)

What diagnoses would you consider at this time? List up to two.

1. _____________________________________________________________

2. _____________________________________________________________

QUESTION 7 (CASE 3)

What additional aspects of history would be most important to know?

Select up to four.

1. Alcohol use

2. Complete psychiatric history

3. Family history of Alzheimer disease

4. Financial status

5. History of fever

6. History of incontinence

7. History of osteoarthritis

8. History of Parkinson disease

9. History of previous CVA

10. Immunization status

11. Marital status

12. Medication history

13. Occupational history

14. Pattern of symptoms over the course of the day, e.g. fluctuations

15. Presence of hallucinations

16. Previous history of depression

17. Previous seizure disorder

18. Previous surgery

19. Recent travel

20. Smoking history

QUESTION 8 (CASE 3)

The interview confirms the daughter's description of the mother.

Physical exam reveals: BP 120/80 supine and 100/70 sitting, RR=22,

P=104, T=38.7ºC. JVP is at the sternal angle. Respiratory exam reveals

dullness, increased tactile fremitus, crackles and bronchial breath

sounds all in the left base. Heart sounds are normal. The abdominal

exam is normal. CNS exam does not reveal any focal findings. What

would you include in your initial investigations? Select up to four.

1. ALT

2. AST

3. Blood cultures

4. B12

5. CBC

6. Chest x-ray

7. CPK

8. CT scan of head

9. Drug screen

10. EEG

11. Electrolytes

12. HIV test

13. MRI of head

14. Protein electrophoresis

15. PT and PTT

16. RBC folate

17. SPECT scan

18. TSH

19. Urea and creatinine

20. VDRL

32 Sample Questions Key Feature Questions Toronto Notes 2005

Toronto Notes 2005 Key Feature Questions Sample Questions 33

CASE 4

A57 year-old man presents to the Emergency Department with a 12

hour history of left flank pain.

QUESTION 9 (CASE 4)

What diagnoses would you consider at this time?

Select up to four.

1. Ruptured abdominal aortic aneurysm

2. Bowel obstruction

3. Appendicitis

4. Pyelonephritis

5. Diverticulitis

6. Renal calculi

7. Biliary colic

8. Renal cell carcinoma

9. Pneumonia

QUESTION 10 (CASE 4)

On history and physical exam, what symptoms and signs might aid in

diagnosis? Choose up to seven.

1. Fever/chills

2. Weight loss

3. Rebound tenderness

4. Costovertebral angle tenderness

5. Urgency and frequency

6. Pruritus

7. Hypotension

8. Pulsatile abdominal mass

9 Nausea and vomiting

10. Left flank mass

11. Productive cough

12. Abdominal guarding

13. Pale stools and dark urine

14. Hematuria

QUESTION 11 (CASE 4)

What tests might be necessary to confirm your diagnosis?

Choose up to six.

1. ESR

2. Serum amylase

3. Urine R&M

4. Chest x-ray

5. Abdominal/pelvic ultrasound

6. 3 views of the abdomen

7. Urine C&S

8. Abdominal CT

9. IVP

10. Total bilirubin

11. Electrolytes

12. 24 hour creatinine clearance

13. KUB

14. Serum Mg

15. Serum alkaline phosphatase

16. Barium enema

17. Serum Ca

18. CBC

CASE 5

A30 year-old woman comes to your office with a 3 day history of vulvar

and vaginal pruritus.

QUESTION 12 (CASE 5)

What diagnosis would you consider at this time? List up to three.

1. _____________________________________________________________

2. _____________________________________________________________

3. _____________________________________________________________

QUESTION 13 (CASE 5)

What features on the history would you inquire about to make a diagnosis?

Choose up to seven.

1. Date of last menstrual period

2. Sexual history

3. Vaginal discharge

4. Vulvar erythema

5. Vaginal odor

6. Recent antibiotic therapy

7. Diabetes

8. Macular rash

9. Psychiatric history

10. Headache

11. Allergies

12. Dysuria

13. Use of douches

14. Sore throat

15. Previous history of STDs

QUESTION 14 (CASE 5)

What tests would you do to confirm the diagnosis? Choose up to four.

1. Urine C&S

2. Wet prep of vaginal discharge for microscopy

3. Urinalysis

4. CBC

5. ESR

6. Culture for gonorrhea

7. Culture for chlamydia

8. Abdominal CT

9. Pelvic ultrasound

10. Culture for Trichomonas vaginalis

CASE 6

A65 year-old man in the emergency department has RUQ pain. He is

otherwise healthy.

QUESTION 15 (CASE 6)

List three diagnoses you should consider in this patient.

1. _____________________________________________________________

2. _____________________________________________________________

3. _____________________________________________________________

QUESTION 16 (CASE 6)

What are the most important questions to ask about on history?

Choose up to seven.

1. Pruritus

2. Pale stools

3. Fever/chills

4. Hematuria

5. Jaundice

6. Anorexia

7. Dark urine

8. Weight loss

9. Back pain

10. Diarrhea

11. Alcohol/drug history

12. Nausea and vomiting

13. Fatigue

14. Cold intolerance

CASE 7

A60 year-old woman comes to your office because she has seen bright

red blood in the toilet bowl after bowel movements for the past month.

She is otherwise healthy.

QUESTION 17 (CASE 7)

What are three common diagnoses you should consider?

1. _____________________________________________________________

2. _____________________________________________________________

3. _____________________________________________________________

QUESTION 18 (CASE 7)

What questions would you ask on history to help determine the cause

of the bleeding? Choose up to five.

1. Family history of inflammatory bowel disease

2. Hematemesis

3. Recent weight loss

4. Nausea and vomiting

5. Change in bowel habits

6. Abdominal pain

7. Fever/chills

8. Family history of colo-rectal cancer

9. Blood mixed with stool or coating the surface

10. Past history of hemorrhoids

11. Dietary history

12. Smoking history

13. Pain on defecation

14. Allergies

CASE 8

You are called to the nursery to assess a 2 hour old term newborn with

respiratory distress. The only information the nurse gives you over the

phone is that the baby was born this morning, had good Apgars and

arrived in the nursery pink on room air and in no distress.

QUESTION 19 (CASE 8)

What are the most common diagnoses that would be in your differential

for this infant? List up to three.

1. _____________________________________________________________

2. _____________________________________________________________

3. _____________________________________________________________

QUESTION 20 (CASE 8)

With respect to the diagnosis, which elements of the history and physical

will be most important in determining the etiology of this baby’s

distress? Select up to four.

1. Delivery mode (C-section, vaginal)

2. Feeding pattern

3. Grunting

4. Head circumference

5. Heart sounds

6. History of maternal diabetes

7. Indrawing

8. Maternal temperature

9. Meconium stained amniotic fluid

10. Oxygen requirement

11. Presence of bruising

12. Presenting part (breech, vertex)

13. Previous obstetric history

14. Tachypnea

15. Vital signs

34 Sample Questions Key Feature Questions Toronto Notes 2005

Toronto Notes 2005 Key Feature Questions Sample Questions 35

CASE 9

A45 year-old migrant farm worker comes to see you with sudden

onset of severe pain in his back and right leg after lifting some bales of

hay. The pain radiates into the lateral aspect of the right foot and is

associated with numbness along the lateral aspect of the foot. He has

had a 5 year history of lower back pain with a similar episode of pain 1

year ago which was concentrated at the posterolateral aspect of the

right calf and lateral aspect of the right foot. It became progressively

worse, then resolved over a 3 week period.

QUESTION 21 (CASE 9)

What elements of the history would you ask about in order to formulate

your diagnosis? Choose up to five.

1. Diffuse muscle cramps

2. Alcohol intake

3. Effect of NSAIDs

4. Family history of back problems

5. Previous history of back injury

6. Shoulder pain

7. Weakness in right leg

8. Allergies

9. Knee problems

10. Urinary incontinence

11. Skin rashes

12. Previous effective physiotherapy

13. Headaches

14. Nocturia

QUESTION 22 (CASE 9)

What elements of the physical exam would you focus on?

Choose up to four.

1. Examination of skin

2. Range of motion of lumbar spine

3. Blood pressure

4. Inspection of muscle bulk in lower extremities

5. Sensory exam of arms

6. Peripheral pulses

7. Reflexes in lower extremities

8. Motor testing of legs

9. Motor testing of arms

10. Sensory exam of legs

11. Bowstring sign

12. Respiratory rate

13. Temperature

14. Straight leg raise test

15. Abdominal exam

16. Rectal exam

17. Pulse

18. Range of motion of knee

19. Auscultation of chest

20. Romberg test

36 Sample Questions Key Feature Scoring Guide Toronto Notes 2005

Key Feature Scoring Guide

QUESTION 1,CASE 1

1. Maximum Number to be selected: = 5

2. The Number you selected (S): = ___

3. Is “S” greater than the Maximum? YES Your Score = 0.

NO Continue.

4. There are 5 correct answers: 2, 4, 8, 13, 20

5. Number of correct answers you selected: = ___

QUESTION 2,CASE 1

1. Maximum Number to be selected: = 5

2. The Number you selected (S): = ___

3. Is “S” greater than the Maximum? YES Your Score = 0.

NO Continue.

4. There are 5 correct answers: 9, 13, 14, 18, 20

5. Number of correct answers you selected: = ___

QUESTION 3,CASE 2

1. Maximum Number to be listed: = 4

2. The Number you listed (L): = ___

3. Is “L” greater than the Maximum? YES Your Score = 0.

NO Continue.

4. There are 8 correct answers:

1. Prematurity

2. Difficult child

3. History of parental child abuse

4. History of substance abuse

5. Social isolation

6. Developmental delay

7. Poverty

8. Parental character/personality disorder

5. Number of correct answers you listed: = ___

QUESTION 4,CASE 2

1. Maximum Number to be selected: = 4

2. The Number you selected (S): = ___

3. Is “S” greater than the Maximum? YES Your Score = 0.

NO Continue.

4. There are 4 correct answers: 7, 9, 14, 19

5. Number of correct answers you selected: = ___

QUESTION 5,CASE 2

1. Maximum Number to be selected: = 4

2. The Number you selected (S): = ___

3. Is “S” greater than the Maximum? YES Your Score = 0.

NO Continue.

4. There are 4 correct answers: 6, 13, 16, 19

5. Number of correct answers you selected: = ___

QUESTION 6,CASE 3

1. Maximum Number to be listed: = 2

2. The Number you listed (L): = ___

3. Is “L” greater than the Maximum? YES Your Score = 0.

NO Continue.

4. There are 2 correct answers:

1. Delirium

2. Acute Confusional State

5. Number of correct answers you listed: = ___

QUESTION 7,CASE 3

1. Maximum Number to be selected: = 4

2. The Number you selected (S): = ___

3. Is “S” greater than the Maximum? YES Your Score = 0.

NO Continue.

4. There are 5 correct answers: 1, 5, 12, 14, 15

5. Number of correct answers you selected: = ___

QUESTION 8,CASE 3

1. Maximum Number to be selected: = 4

2. The Number you selected (S): = ___

3. Is “S” greater than the Maximum? YES Your Score = 0.

NO Continue.

4. There are 5 correct answers: 3, 5, 6, 11, 19

5. Number of correct answers you selected: = ___

Toronto Notes 2005 Key Feature Scoring Guide Sample Questions 37

QUESTION 9,CASE 4

1. Maximum Number to be selected: = 4

2. The Number you selected (S): = ___

3. Is “S” greater than the Maximum? YES Your Score = 0.

NO Continue.

4. There are 4 correct answers: 1, 4, 6, 8

5. Number of correct answers you selected: = ___

QUESTION 10,CASE 4

1. Maximum Number to be selected: = 7

2. The Number you selected (S): = ___

3. Is “S” greater than the Maximum? YES Your Score = 0.

NO Continue.

4. There are 8 correct answers: 1, 2, 4, 5, 7, 8,

10, 14

5. Number of correct answers you selected: = ___

QUESTION 11,CASE 4

1. Maximum Number to be selected: = 6

2. The Number you selected (S): = ___

3. Is “S” greater than the Maximum? YES Your Score = 0.

NO Continue.

4. There are 7 correct answers: 3, 5, 7, 8, 9, 13, 18

5. Number of correct answers you selected: = ___

QUESTION 12,CASE 5

1. Maximum number to be selected = 3

2. The number you listed (L): = ___

3. Is “L” greater than the Maximum? YES Your Score = 0.

NO Continue.

4. There are 4 possible answers:

1. Bacterial vaginosis

2. Vaginal/vulvar candidiasis

3. Trichomonas vaginalis

4. Chemical vaginitis

5. Number of correct answers you listed: = ___

QUESTION 13,CASE 5

1. Maximum Number to be selected: = 7

2. The Number you selected (S): = ___

3. Is “S” greater than the Maximum? YES Your Score = 0.

NO Continue.

4. There are 8 correct answers: 2, 3, 5, 6, 7, 12,

13, 15

5. Number of correct answers you selected: = ___

QUESTION 14,CASE 5

1. Maximum Number to be selected: = 4

2. The Number you selected (S): = ___

3. Is “S” greater than the Maximum? YES Your Score = 0.

NO Continue.

4. There are 4 correct answers: 2, 6, 7, 10

5. Number of correct answers you listed: = ___

QUESTION 15,CASE 6

1. Maximum Number to be listed: = 3

2. The Number you listed (L): = ___

3. Is “L” greater than the Maximum? YES Your Score = 0.

NO Continue.

4. There are 3 correct answers:

1. Biliary colic

2. Acute cholecystitis

3. Bile duct carcinoma

5. Number of correct answers you listed: = ___

QUESTION 16,CASE 6

1. Maximum Number to be selected: = 7

2. The Number you selected (S): = ___

3. Is “S” greater than the Maximum? YES Your Score = 0.

NO Continue.

4. There are 7 correct answers: 1, 2, 3, 5, 6, 7, 8

5. Number of correct answers you selected: = ___

38 Sample Questions Key Feature Scoring Guide Toronto Notes 2005

QUESTION 17,CASE 7

1. Maximum Number to be listed: = 3

2. The Number you listed (L): = ___

3. Is “L” greater than the Maximum? YES Your Score = 0.

NO Continue.

4. There are 5 correct answers:

1. Hemorrhoids

2. Anal fissure

3. Colon or rectal cancer

4. Proctitis or IBD or colitis

5. Diverticulosis

5. Number of correct answers you listed: = ___

QUESTION 18,CASE 7

1. Maximum Number to be selected: = 5

2. The Number you selected (S): = ___

3. Is “S” greater than the Maximum? YES Your Score = 0.

NO Continue.

4. There are 6 correct answers: 1, 5, 8, 9, 10, 13

5. Number of correct answers you selected: = ___

QUESTION 19,CASE 8

1. Maximum Number to be listed: = 3

2. The Number you listed (L): = ___

3. Is “L” greater than the Maximum? YES Your Score = 0.

NO Continue.

4. There are 4 correct answers:

1. Transient tachypnea of the newborn or wet lung

2. Meconium aspiration

3. Pneumonia

4. Cardiac disease

5. Number of correct answers you listed: = ___

QUESTION 20,CASE 8

1. Maximum Number to be selected: = 4

2. The Number you selected (S): = ___

3. Is “S” greater than the Maximum? YES Your Score = 0.

NO Continue.

4. There are 4 correct answers: 1, 6, 8, 9

5. Number of correct answers you selected: = ___

QUESTION 21,CASE 9

1. Maximum Number to be selected: = 5

2. The Number you selected (S): = ___

3. Is “S” greater than the Maximum? YES Your Score = 0.

NO Continue.

4. Essential answers are: 7 and 10

5. Did you select both essential answers?

NO Your Score = 0.

YES Continue.

6. There are 5 correct answers: 3, 5, 7, 10, 12

7. Number of correct answers you listed: = ___

QUESTION 22,CASE 9

1. Maximum Number to be selected: = 6

2. The Number you selected (S): = ___

3. Is “S” greater than the Maximum? YES Your Score = 0.

NO Continue.

4. There are 8 correct answers: 2, 4, 7, 8, 10, 11,

14, 16

5. Number of correct answers you selected: = ___

2 comments:

  1. This comment has been removed by the author.

    ReplyDelete
  2. Thank you so much for sharing this MCCEE Exam Preparation
    article here. I have no words for this great piece of information. I really like it. Thanks for sharing.

    ReplyDelete